AYJR

You might also like

Download as pdf or txt
Download as pdf or txt
You are on page 1of 122

AYJR 2024 (January) – Morning Shift Are You JEE Ready (AYJR)

Questions with Answer Keys and Solutions MathonGo

Q1

A bar magnet having magnetic moment of 2 × 104 J T–1 is free to rotate in a horizontal plane. A horizontal magnetic
field B = 6 × 10–4 T exists in the space. The work done in taking the magnet slowly from a direction parallel to the
field to a direction 60o from the field is

(1) 12 J

(2) 6 J ✅
(3) 2 J

(4) 0.6 J

Answer: (2)

Solution:
Here, M = 2 × 104 J T–1
B = 6 × 10–4T, θ1 = 0o, θ2 = 60o
W = MB(cosθ1 – cosθ2) = MB(1 – cos60o)
4 −4 1
W = 2 × 10 × 6 × 10 (1 − )= 6 J.
2

Q2

The correct graph representing the variation of total energy E , kinetic energy K and potential energy U of a satellite
with its distance from the centre of earth is -

(1)

www.mathongo.com
AYJR 2024 (January) – Morning Shift Are You JEE Ready (AYJR)

Questions with Answer Keys and Solutions MathonGo

(2)

(3) ✅

(4)

Answer: (3)

Solution:

Let M & m are the mass of earth and satellite and r be the radius of the orbital.

Total energy of the satellite is,

GM m 1
E = − ⇒ E ∝ −
2r r

Potential energy of the satellite is,

www.mathongo.com
AYJR 2024 (January) – Morning Shift Are You JEE Ready (AYJR)

Questions with Answer Keys and Solutions MathonGo


GM m 1
U = − ⇒ U ∝ −
r r

Kinetic energy of the satellite is,

GM m 1
K = ⇒ K ∝
2r r

E & U are varying negatively with r and K is varying possitively with r.

Q3

Match the units/dimensions in Column-I with the physical quantities in column -II :
Column - I Column - II

2 −1
(a) [ML T ] (p) Impulse

(b) Joule-sec (q) Planck's constant

−1
(c) [MLT ] (r) Angular momentum

(d) Energy per unit (s) Linear momentum

frequency

(1) a → q, r; b → q, r; c → p, s; d → q, r ✅
(2) a → q, s; b → p, s; c → p, r; d → p, r

(3) a → q, r; b → q, r; c → q, s; d → q, r

(4) a → q, s; b → p, s; c → q, r; d → p, r

Answer: (1)

Solution:
Impulse = Change in linear momentum = [M LT −1
]

Angular momentum = Moment of linear momentum


= [L] [M LT
−1
] = [M L T
2 −1
] = dimension of plank's constant

Q4

A bead of mass m can slide without friction on a fixed circular horizontal ring of radius 3R having a centre at the point
C . The bead is attached to one of the ends of spring with spring constant k. The natural length of spring is R and the
other end of the spring is fixed at point O as shown in the figure. If the bead is released from position A, then the

www.mathongo.com
AYJR 2024 (January) – Morning Shift Are You JEE Ready (AYJR)

Questions with Answer Keys and Solutions MathonGo


kinetic energy of the bead when it reaches point B is

(1) 25

2
kR
2

(2) 9
kR
2


2

(3) 8kR 2

(4) 12kR 2

Answer: (3)

Solution:

Given,
Mass of bead = m
The radius of circular horizontal ring = 3R
The natural length of spring = R
According to the conservation of energy,
KEi + PEi = KEf + PEf

1 2 1 2
⇒ 0 + k[OA − R] = KEf + k[OB − R]
2 2
1 2 1 2
⇒ 0 + k[5R − R] = KEf + k[R − R]
2 2

2
⇒ KEf = 8kR

Q5

Photoelectric effect supports quantum nature of light because


1. There is minimum frequency of light below which no photoelectrons are emitted.

www.mathongo.com
AYJR 2024 (January) – Morning Shift Are You JEE Ready (AYJR)

Questions with Answer Keys and Solutions MathonGo


2. Electric charge of photoelectrons is quantized.
3. Maximum kinetic energy of photoelectrons depends only on the frequency of light and not on its intensity.
4. Even when metal surface is faintly illuminated the photoelectrons leave the surface immediately.

(1) 1, 2, 3

(2) 1, 2, 4

(3) 2, 3, 4

(4) 1, 3, 4 ✅
Answer: (4)

Solution:
Photoelectric effect supports quantum nature of light because
1. There is a minimum frequency of light below which no photoelectrons are emitted.
2. Maximum kinetic energy of photoelectrons depends only on the frequency of light and not on its intensity.
3. Even when the metal surface is faintly illuminated the photoelectrons leave the surface immediately.

Q6

Eight resistances each of resistance 5 Ω are connected in the circuit as shown in the figure. The equivalent resistance
between A and B is –

(1) 8

3
Ω ✅
(2) 16

3
Ω

(3) 15

7
Ω

(4) 19

2
Ω

Answer: (1)

www.mathongo.com
AYJR 2024 (January) – Morning Shift Are You JEE Ready (AYJR)

Questions with Answer Keys and Solutions MathonGo


Solution:
From the symmetry current in AO will be same as current in OB
Simiarly current CO equal to current in OD
The given circuit can be redrawn as

1 1 1 1
= + +
RAB 40 10 5
3

8
∴ RAB = Ω
3

Q7

Let g be the acceleration due to gravity at the earth's surface and K the rotational kinetic energy of the earth. Which of
the following statement is correct if Earth's radius decreases by 2%?

(1) g increases by 2% and K increases by 2%

(2) g decreases by 4% and K decreases by 2%

(3) g increases by 4% and K increases by 4% ✅


(4) g decreases by 2% and K decreases by 4%

Answer: (3)

Solution:

GM
g =
2
R

or g ∝ R
−2

Δg
∴ % change in g is g
× 100 = −2(
ΔR
× 100)
R

= −2 ×(−2)= +4%

Rotational kinetic energy,

www.mathongo.com
AYJR 2024 (January) – Morning Shift Are You JEE Ready (AYJR)

Questions with Answer Keys and Solutions MathonGo


2
L
K = . . .(1)
2I

L = angular momentum = constant

2 2
I = mR . . .(2)
5

From equations (i) and (ii),

−2
K ∝ R

∴ % change in K is ΔK
× 100 = −2(
ΔR
× 100)
K R

= −2 ×(−2)= +4%

Q8

A metal rod moves at a constant velocity in a direction perpendicular to its length. A constant, uniform magnetic field
exists in space in a direction perpendicular to the rod as well as its velocity. Select the correct statement from the
following :

(1) The entire rod is at the same electric potential

(2) There is an electric field in the rod ✅


(3) The electric potential is highest at the center of the rod and decreases towards its ends

(4) The electric potential is lowest at the center of the rod and increases towards its ends

Answer: (2)

Solution:

A metallic rod is moving in uniform magnetic field

www.mathongo.com
AYJR 2024 (January) – Morning Shift Are You JEE Ready (AYJR)

Questions with Answer Keys and Solutions MathonGo

According to Faraday’s law, an induced emf is set up on the rod

εM -εN = BLv

M is at higher potential and N at lower potential.

Thus, an electric field is set up in the rod, ∵ E =


△V

Q9

Figure shows a thin converging lens for which the focal length is 5.00 cm. The lens is in front of a concave spherical
mirror of radius R = 30 cm. If the lens and mirror are 20.0 cm apart and an object is placed 15 cm to the left of the
lens, determine the approximate distance of the final image from lens. (in cm ).

(1) 5.3 cm ✅
(2) 4.6 cm

(3) 6.1 cm

www.mathongo.com
AYJR 2024 (January) – Morning Shift Are You JEE Ready (AYJR)

Questions with Answer Keys and Solutions MathonGo


(4) 12.7 cm

Answer: (1)

Solution:

1 1 1
− =
v −15 5

1 1 1
= −
v 5 15

v = 7.5

v = −12.5

1 1 1
= =

v 12.5 −15

1 2 1
= −

v 25 15

1 12 − 10
=

v 150

v = 75 cm
′′
v = −95 cm

1 1 1
+ =
′′
v 95 5
′′
v = 5.3 cm

Q10

When the arrangement consisting of a horizontal thick copper wire of length 2L and radius 2R, one end of which is
welded to an end of another horizontal thin copper wire of length L and radius R is stretched by applying forces at two
ends. Ratio of the elongation in the thick wire to that in the thin wire at equilibrium is

(1) 0. 50 ✅
(2) 0. 25

(3) 4. 00

(4) 2. 00

Answer: (1)

Solution:

FL
Δℓ =
AY

www.mathongo.com
AYJR 2024 (January) – Morning Shift Are You JEE Ready (AYJR)

Questions with Answer Keys and Solutions MathonGo


2
Δℓ1 L1 A2 2L πR 1
= = × =
Δℓ2 A1 L2 2 2
L
π(2R)

Q11

Select the output Y of the combination of gates shown in figure for inputs A = 1, B = 0; A = 1, B = 1 and
A = 0, B = 0 respectively.

(1) (0, 1, 1)

(2) (1, 0, 1)

(3) (1, 1, 1)

(4) (1, 0, 0) ✅
Answer: (4)

Solution:

According to the given question,

Truth table

¯¯¯¯¯¯¯¯¯¯¯¯ ¯¯¯¯¯¯¯¯¯¯¯¯ ¯¯¯¯¯¯¯¯¯¯¯¯ ¯¯


¯¯¯
¯¯¯
¯¯¯
¯¯
A B C = A ⋅ B D = A ⋅ C E = C ⋅ B Y = D ⋅ E

1 0 1 0 1 1
1 1 0 1 1 0
0 0 1 1 1 0
Therefore, the output, Y =(1, 0, 0)

www.mathongo.com
AYJR 2024 (January) – Morning Shift Are You JEE Ready (AYJR)

Questions with Answer Keys and Solutions MathonGo


Q12

A charged particle q is placed at the centre O of a cube of length L (ABCDEFGH). Another same charge q is placed
at a distance L from O. If the total electric flux through the cube is 6ϕ then the value of ϕ is

q
(1) 4ε0

(2)
q

6ε0

q
(3) 2ε0

q
(4) 3ε0

Answer: (2)

Solution:
q
The net flux through the cube due to the external charge is zero and for the charge inside the cube, it is ε0
through all
q
the surfaces. For one surface, it is 6ε0
.

Q13

A plane Electromagnetic Wave of frequency 30 MHz travels in free space along the x-direction. The electric field
component of the wave at a particular point of space and time is E = 6 Vm-1 along y-direction. Its magnetic field
component B at this point would be

(1) 2 × 10 −8
T along z-direction ✅
(2) 6 × 10
−8
T along x-direction

www.mathongo.com
AYJR 2024 (January) – Morning Shift Are You JEE Ready (AYJR)

Questions with Answer Keys and Solutions MathonGo


(3) 2 × 10
−8
T along y-direction

(4) 6 × 10
−8
T along z-direction

Answer: (1)

Solution:
The frequency of Electromagnetic Wave along y−direction
v = 30 MHz

The electric field component of the wave along y-direction.


−1
E = 6 Vm

In Electromagnetic, the ratio of the amplitudes of electric and magnetic field is always constant and it is equal to
velocity of the Electromagnetic Waves.
E
ie, = c
B

or B =
E

c
=
6

8
3 × 10

or B = 2 × 10
−8
T

Q14

The root-mean-square (RM S) speed of oxygen molecules (O 2) at a certain temperature T (on Kelvin scale) is V . If
the temperature is doubled and oxygen gas dissociates into atomic oxygen, the RM S speed becomes

(1) two times ✅


(2) three times

(3) four times

(4) unchanged

Answer: (1)

www.mathongo.com
AYJR 2024 (January) – Morning Shift Are You JEE Ready (AYJR)

Questions with Answer Keys and Solutions MathonGo


Solution:
3RT
VRMS = √
M

T
VRMS ∝ √ . . . (i)
M

When oxygen gas dissociates into atomic oxygen, its


molar mass M will become half when temperature is doubled.
So, from Eq. (i) VRMS will become two times.
2T

√ M

V 2
RMS
=
VRMS T

M

= √4

= 2


⇒ V = 2VRMS
RMS

Q15

A disc is performing pure rolling on a smooth stationary surface with constant angular velocity as shown in the figure.
At any instant, for the lowermost point of the disc.

(1) Velocity is v, acceleration is zero

(2) Velocity is zero, acceleration is zero


2

(3) Velocty is v, acceleration is


v


R

(4) Velocity is zero, acceleration is non zero

Answer: (4)

Solution:
From figure

www.mathongo.com
AYJR 2024 (January) – Morning Shift Are You JEE Ready (AYJR)

Questions with Answer Keys and Solutions MathonGo

Vnet (for lowest point) = v − Rω = v − v = 0


2 2

and Acceleration =
v v
+ O =
R R

(Since linear speed is constant)


Hence (D)

Q16

Temperature and volume curves are drawn for two thermodynamic processes. For the first process, it is a straight line
and for the second, it is a rectangular hyperbola. The ratio of work done in the first process to the work done in the
second process is

(1) 2:1 ✅
(2) 3:1

(3) 4:1

(4) 3:2

Answer: (1)

Solution:

The first process is a constant pressure process, hence

www.mathongo.com
AYJR 2024 (January) – Morning Shift Are You JEE Ready (AYJR)

Questions with Answer Keys and Solutions MathonGo


w1 = nR(2T0 - T0) = nRT0

The equation for the second process is


c
T =
V

Hence, P = nRT

V
=
nRc

2
V

2V0
nRT0
∴ w = ∫ PdV =
2
V0

w1
∴ = 2 : 1
w2

∴ 2:1

Q17

A sinusoidal wave with amplitude y is travelling with speed v on a string with linear density ρ. The angular frequency
of the wave is ω. The following conclusions are drawn. Mark the one which is correct. (in particular option, if we are
changing one quantity, assume others are kept constant)

(1) Doubling the frequency doubles the rate at which energy is carried along the string

(2) If the amplitude were doubled, the rate at which energy is carried would be halved

(3) If the amplitude were doubled, the rate at which energy is carried would be doubled

(4) The rate at which energy is carried is directly proportional to the velocity of the wave ✅
Answer: (4)

Solution:
1 2 2
Par = μvω A
2

So P ∝ v

2
∝ ω

2
∝ A

So if frequency or amplitude is doubled power will be quadrupled, but if velocity is doubled power will also be
doubled.

Q18

www.mathongo.com
AYJR 2024 (January) – Morning Shift Are You JEE Ready (AYJR)

Questions with Answer Keys and Solutions MathonGo


Current I is flowing in a conductor as shown in the figure. The radius of the curved part is R. The length of straight
portions is very long. The value of the magnetic field at the centre O will be

(1)
μ0 I

4πR
(

2
+ 1) ✅
(2)
μ0 I 3π
( − 1)
4πR 2

(3)
μ0 I π
( + 1)
4πR 2

μ0 I
(4) 4πR
(
π

2
− 1)

Answer: (1)

Solution:
From the figure, the circular part subtended angle at the centre of the circle = 3π/2
∴ Magnetic field at O due to the circular part,
μ0
I θ
B1 = ⋅ ⋅
2 R 2π

μ0 3π/2 μ0
=
2

I

R


=
8

3I

R
...(i)
∵ Point O at the axial position of wire CD
∴ It will not produce magnetic field at O.
∵ Point O is at the end of the wire AE.
∴ Magnetic field due to wire AE,
μ0
B2 =


I

R
...(ii)
Thus, B net = B1 + B2

Putting the value of B and B from equations (i) and (ii), we get
1 1

3μ0 I μ0 I μ0 3π
I
= + = ⋅ [ + 1]
8R 4πR 4π R 2

Q19

What is likely to be orbit number for a circular orbit of diameter 20 nm of the hydrogen atom, if we assume Bohr orbit
to be the same as that represented by the principal quantum number?

(1) 10

(2) 14 ✅
www.mathongo.com
AYJR 2024 (January) – Morning Shift Are You JEE Ready (AYJR)

Questions with Answer Keys and Solutions MathonGo


(3) 12

(4) 16

Answer: (2)

Solution:

Diameter of atom = 20 nm

radius = 10 × 10 −9
m [Where, r =
d

2
]

2
n −9
r = 0. 529 A° = 10 × 10 m
2

−9
2 10×10 ×z
n = [Where, z = 1 hydrogen atom]
0.529A°

−9
10×10 ×(1)
=
−10
0.529×10

2
n = 189. 035

n = 14(Approximate) [Where, 'n' is principle quantum number]

Q20

A stone is dropped into a well in which the level of water is h below the top of the well. If v is the velocity of sound,
the time T after dropping the stone at which the splash is heard is given by,

(1) T =
2h

(2) T = √
2h

g
+
h

v

(3) T = √
2h

g
+
h

2v

(4) T = √
h
+
2h

v
2g

Answer: (2)

Solution:

By using second equation of motion,

1 2
s = ut + at
2

Here the stone is dropped then initial velocity, u = 0 and acceleration is acceleration due to gravity g.

www.mathongo.com
AYJR 2024 (January) – Morning Shift Are You JEE Ready (AYJR)

Questions with Answer Keys and Solutions MathonGo


Time taken by the stone to reach the water level,

1 2
⇒ h = 0 + gt
2 1

2h
⇒ t1 = √ . . .(1)
g

At this time splash is produced and as the sound travels with constant speed. So, the time taken by sound to come to
the opening of well,

Distance
⇒ Time =
Speed

h
⇒t2 = . . . . . . .(2)
v

Total time, T = t1 + t2 .

From equations (1) and (2) we get,

2h h
T = √ +
g v

Q21

Using the following data


Mass hydrogen atom = 1. 00783 u
Mass of neutron = 1. 00867 u
Mass of nitrogen atom ( 7
N
14
)= 14. 00307 u
The calculated value of the binding energy of the nucleus of the nitrogen atom ( 7
N
14
) is close to (Mark answer to
nearest integer in MeV)

Answer: 105

Solution:
The binding energy of nucleus may be defined as the energy equivalent to the mass defect of the nucleus.
If Δm is mass defect then according to Einstein's mass-energy relation.
Binding Energy
2 2
= Δ mc = [{Zmp + (A − Z)mn }−M ]c

2
=(7 × 1.00783 + 7 × 1.00867 − 14.00307)c

or BE = 0 .1124 × 931 .5 MeV

or BE = 104. 73 MeV

Q22

www.mathongo.com
AYJR 2024 (January) – Morning Shift Are You JEE Ready (AYJR)

Questions with Answer Keys and Solutions MathonGo


A 10μF capacitor is charged to a potential difference of 50 V and is connected to another uncharged capacitor in
parallel. Now the common potential difference becomes 20 V. What is the capacitance value (in μF) of the second
capacitor?

Answer: 15

Solution:
q1 = 10 × 50 = 500μC, C1 = 10μF, C2 =?, q2 = 0
q1 +q2
As, V =
C1 +C2
q1 +q2 500+0
C1 + C2 = = = 25μF
V 20

C2 = 25 − C1 = 25 − 10 = 15μF

Q23

A ball is projected from the ground at an angle of 45 with the horizontal surface. It reaches a maximum height of
o

120 m and returns to the ground. Upon hitting the ground for the first time, it loses half of its kinetic energy.
Immediately after the bounce, the velocity of the ball makes an angle of 30 with the horizontal surface. The maximum
o

height it reaches after the bounce (in meters) is _____________.

Answer: 30

Solution:

2 2 o
u sin 45
H1 = = 120
2g
2
u
⇒ = 120 . . .(i)
4g

when half of kinetic energy is lost v = u

√2
2
u 2
( ) sin 30°
√2 2
u
H2 = = . . .(ii)
2g 16g

From (i) and (ii),


H1
H2 = = 30 m
4

Q24

Two identical springs (spring constant = k) are connected to two light rigid rods in parallel combination and then a
third spring (identical to the other two) is connected to this combination in series. The combined system is fixed to the
ceiling and a block of mass m is attached as shown in the figure. If the mass is slightly displaced and then let go, then

www.mathongo.com
AYJR 2024 (January) – Morning Shift Are You JEE Ready (AYJR)

Questions with Answer Keys and Solutions MathonGo

the system shall oscillate with a time period of 2π√ Pm

2k
where P is natural number. Find P.

Answer: 3

Solution:
Consider the system as shown in figure below.

Here, k and k are in parallel. k is in series with parallel combination of k and k


2 3 1 2 3.

Effective spring constant of the combination is


( k2 +k3 ) k1
kef f =
k1 +k2 +k3

2k×k 2
= = k
3k 3

∴ Time period of oscillation


m
T = 2π√
kef f

m
= 2π
√ 2
k
3

3m
= 2π√
2k

Q25

2 kg of ice at −20 °C is mixed with 5 kg of water at 20 °C. The water content of the final mixture is (Latent heat of
ice = 80 kcal kg
−1
, the specific heat of water = 1 kcal kg
−1
°C
−1
and specific heat of ice = 0.5 kcal kg
–1
°C
–1
)
(Mark answer in Kg)

www.mathongo.com
AYJR 2024 (January) – Morning Shift Are You JEE Ready (AYJR)

Questions with Answer Keys and Solutions MathonGo


Answer: 6

Solution:

We have, the heat lost by water = msθ, heat gained by ice = MSθ + ML.
From the principle of calorimetry, we have, the heat lost by water=heat gained by ice.

Hence, msθ 1 = MSθ2 + ML .

5 × 1 ×(20 − 0)= 2 × 0. 5 × 20 + x × 80

x = 1 kg

1 kg of ice melts to form water.


Hence, final amount of water 5 kg + 1 kg = 6 kg .

Here, m, s and θ are the mass, specific heat and temperature difference for water respectively.
1

Similarly, M, S and θ are the corresponding quantities for ice. Here, L is the latent heat of fusion.
2

Q26

The upper half of an inclined plane with inclination ϕ is perfectly smooth while the lower half is rough. A body starting
from rest at the top will again come to rest at the bottom if the coefficient of friction for the lower half is given by μ.
Find 4μ if cot ϕ = 4

Answer: 2

Solution:

Let the length of the inclined plane is d. So, length of the upper half and lower half of the inclined plane is d

2
.

Body starts from rest so initial velocity of the body is u = 0 and final velocity becomes v.

Case I:

For upper half smooth incline plane, component of g will be perpendicularly downwards, hence the acceleration along
the inclined plane a = g sin ϕ
Now, v 2
= u
2
+ 2a(
d

2
)

2 d
∴ v = 2(gsinϕ) . . . (i)
2

www.mathongo.com
AYJR 2024 (January) – Morning Shift Are You JEE Ready (AYJR)

Questions with Answer Keys and Solutions MathonGo

Case II:
For lower half rough incline.

Frictional retardation= μ k gcosϕ

∴ Resultant acceleration= g sinϕ − μ k g cosϕ

2 d
∴ 0 = v + 2(gsinϕ − μk gcosϕ) . . . (ii)
2

From the equation (i) and (ii)


0 =2( gsinϕ)
d

2
+ 2g(sinϕ − μk cosϕ)
d

2
or 0 = sinϕ + sinϕ − μ k
cosϕ

or μ k
cosϕ = 2sinϕ

or μ k
= 2tanϕ

1
= 2 × = 0.5
4

4μk = 2

Q27

The value of inductance L for which the current is maximum in a series LCR AC circuit with C = 10 μF and
ω = 1000 s
−1
is (20 × n) mH, n =?

Answer: 5

Solution:
In series LCR, current is maximum at resonance.
∴ Resonant frequency, ω = 1

√LC

∴ ω
2
=
1
or, L =
1

2
LC ω C

Given, ω = 1000 s −1
and C = 10 μF
1
∴ L = = 0.1 H = 100 mH ⇒ n = 5
−6
1000×1000×10×10

Q28

www.mathongo.com
AYJR 2024 (January) – Morning Shift Are You JEE Ready (AYJR)

Questions with Answer Keys and Solutions MathonGo


If masses of the plates are 2m and m and their edges are a and 2a respectively, then the distance between the centre of
mass of the system and the point O is (value of a is 7 m)

Answer: 7

Solution:

a
2m× +m×2a
2
xcm = = a
3m
a
2m× −ma
2
ycm = = 0
3m

So distance of center of mass of the system is a. a = 7 .

Q29

In Young's double-slit experiment, the intensities at two points P and P on the screen are I and I , respectively. If P
1 2 1 2 1

is located at the central bright fringe and P is located at a distance equal to a quarter of fringe width from P , then
I1

2 1
I2

is

Answer: 2

Solution:
P1 is at central maxima so I 1
= 4I0

For P , ϕ =( 2π β π
2
)( )=
β 4 2

π
I2 = I0 + I0 + 2√I0 I0 cos( )= 2I0
2

www.mathongo.com
AYJR 2024 (January) – Morning Shift Are You JEE Ready (AYJR)

Questions with Answer Keys and Solutions MathonGo


I1
Hence, I2
= 2

Q30

In the figure, the ammeter (I ) reads a current of 10 mA, while the voltmeter reads a potential difference of 3 V. The
ammeters are identical, and the internal resistance of the battery is negligible (consider all ammeters and voltmeters as
non-ideal).
The resistance of the ammeter is, m × 10 2
Ω . What is the value of m?

Answer: 1

Solution:

In the given circuit, potential in the lower portion connected to positive terminal will be 4 V and in the upper portion
connected to negative terminal will be 0 V.

Given: Potential difference across voltmeter V = 3 V .

Let R be the resistance of ammeter.


A

Potential difference across ammeter is

1 V − 0 V =(10 mA)RA

RA = 100 Ω

Thus, the value of m = 1.

www.mathongo.com
AYJR 2024 (January) – Morning Shift Are You JEE Ready (AYJR)

Questions with Answer Keys and Solutions MathonGo

Q31

An alkene (A) on ozonolysis gives a mixture of two carbonyl compounds. Mixture on Clemmensen reduction gives
just one alkane (B) . (B) is the lowest alkane which in pure form can not be prepared by standard Wurtz method. (A)
is

(1) MeCH = CHMe

(2) MeCH 2 CH = CMe2 ✅


(3) MeCH 2 CH2 CH2 CH = CEt2

(4) MeCH 2
CH2 CH = C(Me)Et

Answer: (2)

Solution:
Simplest alkane which cannot be prepared by wurtz reaction is propane. So the alkene must be unsymmetrical having
six carbon, which can give two different carbonyl each contain three carbons.

Q32

Consider the following standard electrode potentials (E in volts) in aqueous solution:


o

3+ +
Element M /M M /M
–––––––––
–––––––– ––––––––

Al −1.66 +0.55

Tl +1.26 −0.34

Based on these data, which of the following statement is correct?

(1) Tl is more stable than Al


+ 3+

(2) Al is more stable than Al


+ 3+

(3) Tl 3+
is more stable than Al 3+

(4) Tl is more stable than Al


+ +

Answer: (4)

www.mathongo.com
AYJR 2024 (January) – Morning Shift Are You JEE Ready (AYJR)

Questions with Answer Keys and Solutions MathonGo


Solution:
o o
ΔG = −nFE

if E o
= positive; then ΔG o
= negative ,
For those conversions whose E values are more positive are more favourable.
o

Q33

An electron practically at rest, is initially accelerated through a potential difference of 100 volts. It then has a de
Broglie wavelength = λ Å . It then get retorted through 19 volts and then has a wavelength λ Å . It further retardation
1 2

λ3 −λ2
through 32 volts changes the wavelength to λ . What is the value of 3
λ1
?

(1) 20

41

(2) 10


63

(3)
20

63

(4) 10

41

Answer: (3)

Solution:
h
λ =
√2meV

h h k
λ1 = = =
√2me ( 100 ) 10√2me 10

h
k =
√2me

h k
λ2 = =
√2me ( 81 ) 9

h k
λ3 = =
√2me ( 49 ) 7

k k 2k
− ( )
λ3 −λ2 7 9 63 20
= = =
λ1 k k 63
( )
10 10

Q34

For an exothermic reaction, following two steps are involved.


Step 1. A + B → I (slow)

Step 2. I →AB (fast)

Which of the following graphs correctly represent this reaction ?

www.mathongo.com
AYJR 2024 (January) – Morning Shift Are You JEE Ready (AYJR)

Questions with Answer Keys and Solutions MathonGo


(1)

(2)


(3)

(4)

Answer: (2)

Solution:
In any reaction mechanism the no. of activated complexes showing maxima of potential energies is the no. of steps in

www.mathongo.com
AYJR 2024 (January) – Morning Shift Are You JEE Ready (AYJR)

Questions with Answer Keys and Solutions MathonGo


that mechanism and in between these maxima is a valley where lies a more stable intermediate. There is one reaction
intermediate lying at valley of two maxima representing two steps. Ea for first step should be higher than second step
as first is slow step. These requirements fulfilled by choice (ii) & not choice (i) the choice (iv) is not possible because
that is for endothermic reaction (ΔH + ve)& (ii) is single step equation so not possible.

Q35

What is the final product of the reaction?


(i) BH3 /THF PCC (i) CH3 MgBr

(CH3 ) C = CHCH2 CH3 −−−−−−−→ −


−→ −−−−−−−→
2
+
(ii) H3 O

(1) 2,3-dimethyl pentan-3-ol ✅


(2) 2,4-dimethyl pentan-3-ol

(3) 2,3-dimethyl pentan-2-ol

(4) 2,2-dimethyl pentan-3-ol

Answer: (1)

Solution:
Here the final product is 2,3-dimethyl pentan-3-ol

Q36

The test that is done for the differentiation of primary amines from secondary and tertiary amine is:

(1) Hell-Volhard Zelinsky reaction

(2) Tollen’s reagent

(3) Azo dye test

www.mathongo.com
AYJR 2024 (January) – Morning Shift Are You JEE Ready (AYJR)


Questions with Answer Keys and Solutions MathonGo
(4) Carbylamine test

Answer: (4)

Solution:

The Carbylamine reaction is a chemical test for detection of primary amines. In this reaction, the analyte is heated with
alcoholic potassium hydroxide and chloroform. If a primary amine is present, the isocyanide is formed.

For example, the reaction with ethylamine:

C2 H5 NH2 + CHCl3 + 3KOH → C2 H5 NC + 3KCl + 3H2 O

Ethylamine Chloroform Ethyl isocyanide

Q37

25 mL of household bleach solution was mixed with 30 mL of 0.50 M KI and 10 mL of 4 N acetic acid. In the
titration of the liberated iodine, 48 mL of 0.25 N Na 2
S2 O3 was used to reach the end point. The molarity of the
household bleach solution is

(1) 0.24 M ✅
(2) 0.48

(3) 0.024 M

(4) 0.96 M

Answer: (1)

Solution:
H2 O2 + 2I

+ 2H
+
→ I2 + 2H2 O ...(i)
Bleach 0.5M (from CH3 COOH)
25 ml 30 ml
4 N(10 ml)

I2 + 2Na2 S2 O3 → Na2 S4 O6 + 2NaI ...(ii)


0.25 N(48 ml)

m. mol of I 2
=
1

2
(m. Moles of Na 2
S2 O3 )
1
= × (0. 25 × 48) = 6 m. mol
2

Using equation (i)


1 m. Mol of I 2 ≡ 1 m. Mol of H 2 O2

m.mol of H 2 O2 = 6 m.mol
Molarity of H 2 O2 =
6

25
= 0.24M

www.mathongo.com
AYJR 2024 (January) – Morning Shift Are You JEE Ready (AYJR)

Questions with Answer Keys and Solutions MathonGo


Q38

(1)


(2)

(3)

(4)

www.mathongo.com
AYJR 2024 (January) – Morning Shift Are You JEE Ready (AYJR)

Questions with Answer Keys and Solutions MathonGo


Answer: (1)

Solution:

Cl¯ is –I > +M but ortho/para directing


–M group is at leaving group to ortho present so ArSN2 will takes place.

Q39

What is the freezing point of a solution that contains 10.0 gof a glucose (C 6
H12 O6 ) in 100 g of H 2 O? Kf =1.86°C/m .

(1) −0.186°C

(2) +0.186°C

(3) −0.10°C

(4) −1.03°C ✅
Answer: (4)

Solution:
WB
ΔTf = Kf m = Kf
MB .WA
−1
1.86 K kg mol ×10.0 g
=
−1 1 kg
180 g mol ×100 g× ( )
1000 g

1.86×10.0×1000 K
=
180×100

= 1.03 K = 1.03°C

F.P. of solution = F. P of water −ΔT f

= 0°C − 1.03°C = −1.03°C

Q40

The major product of the reaction is

www.mathongo.com
AYJR 2024 (January) – Morning Shift Are You JEE Ready (AYJR)

Questions with Answer Keys and Solutions MathonGo


(1)

(2)

(3)


(4)

Answer: (3)

Solution:

Q41

2-propanol will be the product of which one of the following reactions? Multiple can be true.
+
H
I. CH2 = CH − CH3 + H2 O −
−→

www.mathongo.com
AYJR 2024 (January) – Morning Shift Are You JEE Ready (AYJR)

Questions with Answer Keys and Solutions MathonGo


( i ) CH3 Mgl

II. CH3 − CHO −−−−−−→


+
( ii ) H3 O

( i ) C2 H5 Mgl

III. CH2 O −−−−−−−→


+
( ii ) H3 O

Neutral KMnO4

IV. CH2 = CH − CH3 −−−−−−−−→

(1) I and II ✅
(2) II and III

(3) I and III

(4) II and IV

Answer: (1)

Solution:

Q42

The standard enthalpy of formation (ΔH °f ) at 298 K for methane, CH 4 (g) is − 78.4 kJ mol
−1
. The additional
information required to determine the average energy for C − H bond formation would be

(1) The dissociation energy of H and enthalpy of sublimation of carbon


2 ✅
(2) Latent heat of vaporisation of methane

(3) The first four ionisation energies of carbon and electron gain enthalpy of hydrogen

www.mathongo.com
AYJR 2024 (January) – Morning Shift Are You JEE Ready (AYJR)

Questions with Answer Keys and Solutions MathonGo


(4) The dissociation energy of hydrogen molecule, H 2

Answer: (1)

Solution:
Carbon is found in solid state. The state of substance affects the enthalpy change.

C(s) → C(g) Sublimation

and H 2
(g)→ 2H(g) dissociation are required for C − H bond.

Q43

Because of lanthanoid contraction, which of the following pairs of elements have nearly same atomic radii? (Numbers
in the parenthesis are atomic numbers).

(1) Zr (40) and Nb (41)

(2) Zr (40) and Hf (72) ✅


(3) Zr (40) and Ta (73)

(4) Ti (22) and Zr (40)

Answer: (2)

Solution:
Due to lanthanoid contraction (poor screening effect caused by inner d and f orbitals electrons due to their deeper
inside location) atomic radii of Zr and Hf is almost similar.
Zr = 159 pm, Hf = 156 pm

Q44

Match List - I with List - II. List II indicate magnetic moment of central metal atom.

List-I List-II
(a) (i)
3−
[Fe (CN) ] 5. 92 BM
6

(b) (ii)
3+
[Fe (H2 O) ] 0 BM
6

(c) [Fe (CN) ]4−


6
(iii) 4. 90 BM

2+
(d) [Fe (H2 O) ]
6
(iv) 1. 73 BM

Choose the correct answer from the options given below.

www.mathongo.com
AYJR 2024 (January) – Morning Shift Are You JEE Ready (AYJR)

Questions with Answer Keys and Solutions MathonGo


(1) (a)-(iv), (b)-(ii), (c)-(i), (d)-(iii)

(2) (a)-(ii), (b)-(iv), (c)-(iii), (d)-(i)

(3) (a)-(i), (b)-(iii), (c)-(iv), (d)-(ii)

(4) (a)-(iv), (b)-(i), (c)-(ii), (d)-(iii) ✅


Answer: (4)

Solution:

[Fe(CN)6 ]
3−
has Fe 3+
central ion (d
5
)

Electronic configuration (SFL)


0,0
2,2,1
t2g eg

n = 1

Magnetic moment:

μ = √n(n + 2) BM

= √1(1 + 2)

= √3

= 1. 73 BM

3+
[Fe (H2 O) ]
6
: has d electrons.
5

Electronic configuration (WFL)


1,1
1,1,1
t2g eg

n = 5

μ = √5(5 + 2) = √35

= 5. 92 B. M.

has d electrons.
4− 6
[Fe(CN)6 ]

Electronic configuration (SFL)


2,2,1 0,0
t eg
2g

n = 0

μ = 0 B. M.

www.mathongo.com
AYJR 2024 (January) – Morning Shift Are You JEE Ready (AYJR)

Questions with Answer Keys and Solutions MathonGo


2+
[Fe (H2 O) ] has d electron
6
6

Electronic configuration: (WFL)


2,2,1 1,1
t eg
2g

n = 4

μ = √4(4 + 2) = 4. 90 B. M.

Q45

Which of the following statement is true about hydrogen bonding?

(1) Cl and N have comparable electronegativities yet there is no H-bonding in HCl because size of Cl is large ✅
(2) Intermolecular H-bonding results in decrease in melting point and boiling point

(3) Ice has maximum density at 0° C due to H-bonding

(4) The magnitude of hydrogen bonding is more in gaseous HF than in liquid HF

Answer: (1)

Solution:
For hydrogen bonding, the size of the atom should be small and electronegativity must be high for e.g. F, O, N.
Although chlorine has high electronegativity but due to its larger size hydrogen bonding is not possible with it. The
magnitude of hydrogen bonding is more in gaseous HF than in liquid HF, this is wrong because, in the gaseous form,
the force of attraction is verv weak.

Q46

In the given reaction,

,
the major product 'X' will be

www.mathongo.com
AYJR 2024 (January) – Morning Shift Are You JEE Ready (AYJR)

Questions with Answer Keys and Solutions MathonGo


(1)


(2)

(3)

(4)

Answer: (1)

Solution:
It is a case of intramolecular aldol condensation and the reaction is as follows

Q47

The simultaneous solubility of AgCN (K sp = 2.5 × 10


−6
) and AgCl (K sp = 1.6 × 10
−10
) in 1 M NH (aq) are
3

respectively
Given K f [Ag(NH3 )2 ]
+
= 10
7

www.mathongo.com
AYJR 2024 (January) – Morning Shift Are You JEE Ready (AYJR)

Questions with Answer Keys and Solutions MathonGo


(1) 1.58 × 10 −3
, 1.26 × 10
−5

(2) 0.04, 6.25 × 10 −8

(3) 5.58 × 10 −8
, 0.037

(4) 0.037, 5.78 × 10 −8



Answer: (4)

Solution:
+ −
AgCl(s) + 2NH3 (aq) ⇌ Ag(NH3 ) + Cl (aq)
2
0−2x x x

−10 7 −3
K1 = 1.6 × 10 × 10 = 1.6 × 10
+ −
[Ag(NH3 ) [Cl ]
2
K1 =
2
[NH3 ]


[Cl ] −3
1.6×10 5

= = 6.4 × 10
[CN ] −9
2.5×10
2
x −3
K1 = = 1.6 × 10
2
(1−2x)

x
= 0.04
(1−2x)

x = 0.037

− 0.037 −8
[CN ] = = 5.78 × 10
5
6.4×10

Q48

Assertion: The majority of noble gas compounds are those of xenon (Xe).

Reason: Xenon has the lowest ionization enthalpy and can be easily oxidized by oxidizing agents like oxygen and
fluorine

(1) Assertion and reason are correct, and reason is the correct explanation of assertion. ✅
(2) Assertion and reason are correct, and reason is not the correct explanation of assertion

(3) Assertion is correct, reason is wrong

(4) Assertion is wrong, reason is correct

Answer: (1)

Solution:

Xe is a noble gas with a large size.

As the size increases, the ionization potential decreases, and is easily oxidized by oxidants like oxygen and fluorine

www.mathongo.com
AYJR 2024 (January) – Morning Shift Are You JEE Ready (AYJR)

Questions with Answer Keys and Solutions MathonGo

Q49

Consider the acidity of the carboxylic acids.


(i) PhCOOH
(ii) o − NO 2 C6 H4 COOH

(iii) p − NO 2 C6 H4 COOH

(iv) m − NO 2 C6 H4 COOH

Which of the following order is correct?

(1) i > ii > iii > iv

(2) ii > iv > iii > i

(3) ii > iv > i > iii

(4) ii > iii > iv > i ✅


Answer: (4)

Solution:

In Aromatic carboxylic acid group (Benzoic acid), the acidity of benzoic acid is increased by the presence of an
electron-withdrawing group.

The − NO group is an electron withdrawing group.


2

The − NO group at o−position and p−position (+I effect) has more pronounced electron withdrawing effect than as
2

− NO2 group at m − position (−I effect).

Electron withdrawing group at o−isomer will have higher acidity than corresponding m and p isomer, due to ortho −
effect.

The order of acidity is,

Correct order of acidity is ⇒ ii > iii > iv > i

www.mathongo.com
AYJR 2024 (January) – Morning Shift Are You JEE Ready (AYJR)

Questions with Answer Keys and Solutions MathonGo

Q50

Given the electronic configurations of few elements. Select the incorrect Match:
2 2 5
1. 1s 2s 2p Most electronegative element in the periodic table

2 2 3 rd th
2. 1s 2s 2p An element belonging to 3 period and 5 group

2 2 6 2 6 8 2
3. 1s 2s 2p 3s 3p 3d 4s A d-block element

2 2 6 2 6 th
4. 1s 2s 2p 3s 3p An element from 18 group

(1) 1 and 4 only

(2) 2 only ✅
(3) 1, 2 and 3

(4) All are the incorrect matches.

Answer: (2)

Solution:

Based on the total number electrons we can identify the atomic number of element, group number and period number
in the periodic table.

The electronic configuration 1s 2


2s
2
2p
5
represents the element fluorine. Fluorine is the most electronegative element
in the periodic table, and it belongs to 2nd period.

The electronic configuration 1s 2


2s
2
2p
3
represents the element nitrogen. It belongs to 2 nd
period and 15 group in the
th

periodic table.

The electronic configuration 1s 2


2s
2
2p
6
3s
2 6
3p 3d
8
4s
2
represents the element nickel. It is a d-block element.

The electronic configuration 1s 2


2s
2
2p
6
3s
2 6
3p represents the element argon. It is an element from 18 group. th

Q51

Copper sulphate solution (250 mL) was electrolysed using a platinum anode and a copper cathode. A constant current
of 2 mA was passed for 16 min . It was found that after electrolysis the absorbance of the solution was reduced to 50%
of its original value. Concentration of copper sulphate in the solution to begin with is x × 10 . Find the value of x −5

(mark answer to nearest integer)

Answer: 8

www.mathongo.com
AYJR 2024 (January) – Morning Shift Are You JEE Ready (AYJR)

Questions with Answer Keys and Solutions MathonGo


Solution:
−3

The number of faraday's passed =


2×10 ×16×60

96500

−5
= 1.99 × 10

⇒ number of gram equivalent of Cu 2+


deposited as Cu(s)
−5
= 1.99 × 10

⇒ number of moles of Cu 2+
deposited
1.99 −5 −5
= × 10 ≈ 10
2

Absorbance is directly proportional to [Cu 2+


] .

No. of moles of Cu2+ initially present = 2 x 10-5 moles


2+ −5 1000
[Cu ] = 2 × 10 ×
250
Initial

= 8 x10-5M

Q52

If the pK of acetic acid and pK of NH


a b 4 OH are 4.76 and 4.75 respectively, what will be the pH of ammonium acetate
solution? (mark answer to nearest integer)

Answer: 7

Solution:
When a weak acid (acetic acid) and weak base (NH 4
OH) are mixed to form a solution. pH of the solution is then
determined by following formula:
1
pH = 7 + [pK − pK ]
a b
2

Given, pK a
= 4.76, pK
b
= 4.75

1
pH = 7 + [4.76 − 4.75]
2

pH = 7 + 0.005

pH = 7.005

Q53

How many of the following are obtained on heating Potassium permanganate?

K2 MnO4 , MnO2 , O2 , Mn2 O3

Answer: 3

Solution:
heat

2 KMnO4 −
−−→ K2 MnO4 + MnO2 + O2

www.mathongo.com
AYJR 2024 (January) – Morning Shift Are You JEE Ready (AYJR)

Questions with Answer Keys and Solutions MathonGo


Q54

The hydrated salt Na 2


SO4 ⋅ nH2 O loses all water of crystallization on heating and is reduced to 44. 1% of its original
weight. Therefore, the value of n is (roundoff value to n to nearest integer)

Answer: 10

Solution:

In the given compound, because after heating, molecule loses all water of crystallization, so hydrated salt contains
44. 1% salt and 55. 9% is water.

The molecular mass of Na 2 SO4 = 142 g

The molecular mass of water =18 g

% Composition of salt = Mass


Mass

of
of salt

hydrated salt

Let the hydrated salt contains n number of water molecules.

Molecular mass of hydrated salt = 142 + 18n

142
44. 1 = × 100
142+18n

Solving for n, we get, n = 10

Q55

C5 H10 O is a carbonyl compound. The number of structural isomers possible for this molecular formula are

Answer: 7

Solution:

www.mathongo.com
AYJR 2024 (January) – Morning Shift Are You JEE Ready (AYJR)

Questions with Answer Keys and Solutions MathonGo

Q56

The number of paramagnetic species among the following is


O2 , CO, N2 , C2 , CsO2 , BaO2 , CO2

Answer: 2

Solution:
O2 and CsO are paramagnetic.
2

Q57

During the nuclear explosion, one of the products is 90 Sr with half life of 6.93 years. If 1 μ g of 90 Sr was absorbed in
the bones of a newly born baby in place of Ca, how much time (in years) is required to reduce it by 90%, if it is not
lost metabolically?
Report your answer by rounding it up to a nearest whole number.

Answer: 23

Solution:
100
t90% ln
10
=
t ln 2
50%

ln10
years
6.93×ln 10
t90% = × t50% = = 23.03
ln2 0.693

Q58

The number of peptide bond(s) in the following molecule is

www.mathongo.com
AYJR 2024 (January) – Morning Shift Are You JEE Ready (AYJR)

Questions with Answer Keys and Solutions MathonGo


Answer: 2

Solution:
This compound has 2 peptide bonds.

Q59

Calculate the final temperature of the gas, if one mole of an ideal gas is allowed to expand reversibly and adibatically
from a temperature of 27°C and the work done during the process is 3 kJ .
Given (C V
= 20 J/K) (mark answer in Kelvin)

Answer: 150

Solution:
Given number of moles =1
Initial temperature = 27 o
C = 300 K

Work done by the system = 3 kJ = 3000 K

It will be (−) because work is done by the system.


Heat capacity at constant volume (Cv) = 20 J/k

We know that work done


W = −nCV (T2 − T1 ) ;
3000 = −1 × 20(T2 − 300)

3000 = −20T2 + 6000

3000
20T2 = 3000T2 = = 150 K
20

Q60

www.mathongo.com
AYJR 2024 (January) – Morning Shift Are You JEE Ready (AYJR)

Questions with Answer Keys and Solutions MathonGo


The number of π bonds in the major product will be _____.

Answer: 1

Solution:
LiAlH4 reduces aldehydes, ketones and carboxylic acids to corresponding alcohols and nitro (−NO 2) group to amino
(−NO2 ) group. But LiAlH cannot reduce carbon-carbon double bonds.
4

Q61

dy 1
If = f (x) + ∫
0
f (x)dx , then the equation of the curve y = f (x) passing through (0, 1) is


dx

x
2e −e+1
(1) f (x) = 3−e

(2) f (x) = 3e −2e+1

2(2−e)

(3) f (x) = e −2e+1

e+1

(4) none of these

Answer: (1)

Solution:
′′
f (x)
′′ ′
f (x) = f (x) ⇒ = 1

f (x)

On integrating,
′ x
f (x) = Ce

Which gives f (x) = Ce x


+ D

But f (0) = 1 ⇒ C + D = 1
x
∴ f (x) = Ce + 1 − C

So, f ′
(x) = Ce
x

1
Putting it in f ′
(x) = f (x) + ∫
0
f (x)dx

1 2
x x x
⇒ Ce = Ce + 1 − C + ∫ (Ce + 1 − C) dx ⇒ C =
0 3−e

www.mathongo.com
AYJR 2024 (January) – Morning Shift Are You JEE Ready (AYJR)

Questions with Answer Keys and Solutions MathonGo


x
2e −e+1
So, f (x) = 3−e

Q62

Let S be the set of 6-digit numbers a 1 a2 a3 a4 a5 a6 (all digits distinct)


Where a 1 > a2 > a3 > a4 < a5 < a6 . Then n(S) is equal to

(1) 210

(2) 2100 ✅
(3) 4200

(4) 42

Answer: (2)

Solution:
First, 6 distinct digits can be selected in 10
C6 ways.
Now the position of smallest digit in them is fixed i.e. position 4.
Of the remaining 5 digits, two digits can be selected in 5
C2 ways.
These two digits can be placed to the right of 4 th
position in one way only.
The remaining three digits to the left of 4 th
position are in the required order automatically.
So n(S) = 10
C6 ×
5
C2 = 210 × 10 = 2100

Q63

The mean and variance of 20 observations are found to be 10 and 4 respectively. On rechecking, it was found that an
observation 8 is incorrect. If the wrong observation is omitted, then the correct variance is

(1) 7

(2)
100

19

(3) 1400


361

(4) 1440

361

Answer: (4)

Solution:
We have n = 20 , ¯
¯¯
x old = 10 , V arold = 4
Σxi
¯
¯¯
⇒ x = ⇒ Σxi = 200
20

¯
¯¯ 192
⇒ (Σxi ) = 192, X new =
new 9

www.mathongo.com
AYJR 2024 (January) – Morning Shift Are You JEE Ready (AYJR)

Questions with Answer Keys and Solutions MathonGo


2
Σx 2
i ¯
¯¯
V ar ( old ) = − (x old )
20
2
Σx
i
4 = − 100
20

2
Σx = 2080
i ( old )

2
Σx = 2080 − 64 = 2016
i new

2
2016 192 1440
V ar ( new ) = − ( ) =
19 19 361

Q64

If A and B are matrices with 24 and 40 elements respectively, then the number of possible orders of A and B such that
AB is defined is

(1) 2

(2) 3

(3) 4 ✅
(4) 8

Answer: (3)

Solution:
If AB is defined then order of A must be m × n and order of B is n × p
3
mn = 24 = 2 × 3

3
np = 40 = 2 × 5

n can be 2 0 1
,2 ,2 ,2
2 3

There are only 4 possibilities of n, so the number of possible orders of A, B is 4

Q65

y
The lines L 1 : x = y = z, L2 : x =
2
=
z

3
and a line L is passing through (1, 1, 1) form a triangle of area √6 units,
3

(1, 1, 1) being one of the vertices of the triangle. Then the point of intersection of the L with L is 3 2

(1) (1, 2, 3)

(2) (2, 4, 6) ✅
(3) ( 4

3
,
8

3
, 4)

(4) (1, 5, 7)

Answer: (2)

www.mathongo.com
AYJR 2024 (January) – Morning Shift Are You JEE Ready (AYJR)

Questions with Answer Keys and Solutions MathonGo


Solution:

√6
cos θ =
6
⇒ sin θ = (where θ is the angle between L 1& L2 )
√42 √42

1
Area △OAB = (OA)(OB) sin θ
2

1 √6
= (√3)|λ|(√14) = √6 ⇒ λ = ±2
2 √42

Q66

3 π −1
+cos x

If ∫ , then k is equal to
4 2

1 dx = k
−1 −1 −1
2 sin x+3 cos x+cos (1−x)


4

(1) 0.25

(2) 0.5

(3) 1.5

(4) 1.25

Answer: (1)

Solution:

3 π −1
4 + cos x
2
I = ∫ dx
−1 −1
1 π + cos x + cos (1 − x)
4

−1 −1
(sin x + cos x = π/2)

Apply ∫
b b
f (x)dx = ∫ f (a + b − x)dx
a a
3 π −1
+cos (1−x)
4 2
I = ∫1 dx
−1 −1
π+cos (1−x)+cos x
4

www.mathongo.com
AYJR 2024 (January) – Morning Shift Are You JEE Ready (AYJR)

Questions with Answer Keys and Solutions MathonGo


3
−1 −1
4 π + cos (x) + cos (1 − x)
Add 2I = ∫ dx
−1 −1
1 π + cos (x) + cos (1 − x)
4

3/4 1
= (x) =
1/4
2

1
I = = 0.25
4

Q67

An urn contains 3 red, 4 green and certain number of white balls. Two balls are drawn simultaneously and found to be
of different colour. If the chance that none of them was a white ball is 2/9, then number of white balls is equal to -

(1) 3

(2) 4

(3) 6 ✅
(4) 8

Answer: (3)

Solution:
Let us define some events
A : 1R&1G ball is drawn
B : 1R&1 W ball is drawn
C : 1 W&1G ball is drawn
D : 2 balls drawn are of different colour
Let number of white balls = x
A 2
P( ) =
D 9

From Baye's theorem


A (A ∩ D)
P ( ) =
D P (D)

D
P (A) ⋅ P ( )
A

=
D D D
P (A) ⋅ P ( ) + P (B)P ( ) + P (C)P ( )
A B C

3 4
C1 × C1

x+7
C2
=
3 4 x 3 x 4
C1 × C1 C1 × C1 C1 × C1
+ +
x+7 x+7 x+7
C2 C2 C2

12 2
⇒ = ⇒ 108 = 24 + 14x
12 + 7x 9

⇒ 14x = 84 ⇒ x = 6

www.mathongo.com
AYJR 2024 (January) – Morning Shift Are You JEE Ready (AYJR)

Questions with Answer Keys and Solutions MathonGo


Q68

Let f (x) be defined for all x ∈ R such that lim x→0


[f (x) + ln(1 −
1

f (x)
) − ln(f (x))] = 0 then lim x→0
f (x) is equal
e

to

(1) 0 ✅
(2) 1

(3) e

(4) 1/e

Answer: (1)

Solution:

1
lim [f (x) + ln(1 − ) − ln(f (x))]
x→0 f (x)
e

f (x) f (x)
lim [f (x) + ln(e − 1) − ln e − ln(f (x))]
x→0

f (x)
e − 1
f (x)
lim [ln(e − 1) − ln f (x)] = lim [ln( )]
x→0 x→0 f (x)

= 0 if f (x) → 0 as x → 0

As f (x) is defined ∀x ∈ R
f (0) = 0

Q69

Let f (x) = x 2
+ λx + μ cos x, λ being an integer and μ a real number. The number of ordered pairs (λ, μ) for which
the equations f (x) = 0 and f (f (x)) = 0 have the same (non empty) set of real roots is

(1) 4 ✅
(2) 6

(3) 8

(4) None of these

Answer: (1)

Solution:
Let α be a root of f (x) = 0, so we have f (α) = 0 and thus f (f (α)) = 0,
⇒ f (0) = 0 ⇒ μ = 0.

We then have f (x) = x(x + λ) and thus α = 0, −λ.

www.mathongo.com
AYJR 2024 (January) – Morning Shift Are You JEE Ready (AYJR)

Questions with Answer Keys and Solutions MathonGo


2
f (f (x)) = x(x + λ) (x + λx + λ)

We want λ such that x 2


+ λx + λ has no real roots besides 0 and −λ. We can easily find that 0 ≤ λ < 4.

Q70

If the function f : R → R is defined by f (x) = |x|(x − sin x), then which of the following statements is TRUE?

(1) f is one-one, but NOT onto

(2) f is onto, but NOT one-one

(3) f is BOTH one-one and onto ✅


(4) f is NEITHER one-one NOR onto

Answer: (3)

Solution:
2 sin x
f (∞) = Ltx→∞ x (1 − ) = ∞
x

⇒ Range of f (x) = R
⇒ f (x) is an onto function
−2x + sin x + x cos x, x < 0

f (x) = {
2x − sin x − x cos x, x ≥ 0

For (0, ∞)

f (x) = (x − sin x) + x(1 − cos x)

always +ve always +ve

or 0 or 0

⇒ f (x) > 0


⇒ f (x) ≥ 0, ∀x ∈ (−∞, ∞)

equality at x = 0
⇒ f (x) is one-one function
From (1) & (2), f (x) is both one-one & onto.

Q71

If z 1
2
+ |z1 | = z2
2
+ |z2 | = z3
2
+ |z3 | = 0 (z1 ≠ z2 ≠ z3 ) and z 1, z2 and z are the roots of equation
3

ax
3
+ bx
2
+ x + c = 0 ; then-

(1) a = 1 ✅
(2) b = −1

(3) a = −1

www.mathongo.com
AYJR 2024 (January) – Morning Shift Are You JEE Ready (AYJR)

Questions with Answer Keys and Solutions MathonGo


(4) b = 1

Answer: (1)

Solution:
Consider z 2
= −|z|

|z| = 0 or |z| = 1

2
⇒ z = 0, z = −1, z = ±i

Required equation x (x 2
+ 1) = 0
3
x + x = 0

a = 1, b = 0

Q72

If the image of the point M (λ, λ 2


) on the line x + y = λ is N (0,2), then the sum of the squares of all the possible
2

values of λ is equal to

(1) 5 ✅
(2) 2

(3) 1

(4) 4

Answer: (1)

Solution:

Since the mid-point of the line segment M N lie on the given line,
2
λ λ +2 2
⇒ + = λ … . .(i)
2 2

Also, the line segment M N is perpendicular to the given line


2
λ −2
⇒( )×(−1)= −1 … . .(ii)
λ−0

www.mathongo.com
AYJR 2024 (January) – Morning Shift Are You JEE Ready (AYJR)

Questions with Answer Keys and Solutions MathonGo

As (i) and (ii) must be satisfied simultaneously,


∴ On solving, we get,

λ = −1,2

sum of squares = (−1) 2


+ (2)
2
= 1 + 4 = 5

Q73

The coefficient of x in the expansion of (1 + x)


5 21
+ (1 + x)
22
+ … + (1 + x)
30
is

(1) 51
C5

(2) 31
C5 −
21
C5

(3) 31
C6 −
21
C6 ✅
(4) 30
C5 +
20
C5

Answer: (3)

Solution:
Using sum of GP of 10 terms
21 10
(1+x) [(1+x) −1]

Coefficient of x in 5

(1+x)−1

31 21
(1 + x) − (1 + x)

x
6 31 21
Coefficient of x in (1 + x) − (1 + x)

31 21
= C6 − C6

Q74

www.mathongo.com
AYJR 2024 (January) – Morning Shift Are You JEE Ready (AYJR)

Questions with Answer Keys and Solutions MathonGo


The area (in sq. units) bounded by y = 2 and y = 2x − x from x = 1 to x = 2 is k log
x 2
2
e − m , then the value of ∣∣ k

m

is equal to

(1) 3 ✅
(2) 2

(3) 4

(4) 1

Answer: (1)

Solution:

Required area is
2

x 2
A = ∫ [2 −(2x − x )]dx

x 3 2
2 2 x
= [ − x + ]
ln 2 3
1

4 8 2 1
=( − 4 + )−( − 1 + )
ln 2 3 ln 2 3

2 7
= − 3 +
ln 2 3

2 2 2
= − = 2log2 e −
ln 2 3 3

2
∴ k = 2 & m =
3

k
∣ ∣
⇒∣ ∣= 3
m

www.mathongo.com
AYJR 2024 (January) – Morning Shift Are You JEE Ready (AYJR)

Questions with Answer Keys and Solutions MathonGo


Q75

If (2, 4) is interior point of the circle x 2


+ y
2
− 6x − 10y + λ = 0 and the circle does not cut the coordinate axes, then
λ ∈

(1) (25, 34)

(2) (9, 32)

(3) (25, 32) ✅


(4) (4, 25)

Answer: (3)

Solution:
(x1 , y1 ) lies inside if S 11 < 0

4 + 16 − 12 − 40 + λ < 0 ⇒ λ < 32 ...(i)


The circle cuts the x-axis at the points given by y = 0, x 2
− 6x + λ = 0

Discriminant < 0, 36 − 4λ < 0, λ > 9 ...(ii)


2
x = 0 ⇒ y − 10y + λ = 0

Discriminant < 0, 100 − 4λ < 0, λ > 25 ...(iii)


(i) to (iii) ⇒ λ ∈ (25, 32).

Q76

→, →b and c→ are three mutually perpendicular unit vectors and d→ is a unit vector which makes equal angles with a
If a →, →b
→ →
and c→ then the value of | a
→ → 2
+ b + c + d |

(1) 4 + 2√2

(2) 4 + 2√3 ✅
(3) 2 + √5

(4) 3 + √5

Answer: (2)

Solution:

→ → →
→ → → 2 →
| a + b + c + d | = ∑| a | + 2∑ a ⋅ b

→ →
→ →
= 4 + 2 d ⋅ ( a + b + c )(a, b, c are mutually ⊥ r)

Let d→ = λa
→ + μ→b + vc→
Then d→ ⋅ a
→ = d→ ⋅ →b = d→ ⋅ c→ = cos θ

www.mathongo.com
AYJR 2024 (January) – Morning Shift Are You JEE Ready (AYJR)

Questions with Answer Keys and Solutions MathonGo


∴ λ = μ = v = cos θ

Also λ 2
+ μ
2
+ v
2
= 1 = 3 cos
2
θ or cos θ = 1

√3

→ → 3
→ → 2
∴ | a + b + c + d | = 4 ± 2 ⋅
√3

3
= 4 ± 2
√3

Q77

2
∣ λ + 3λ λ − 1 λ + 3∣
∣ ∣
If pλ 4
+ qλ
3
+ rλ
2 2
+ sλ + t = ∣ λ + 1 2 − λ λ − 3∣ , then p is equal to
∣ ∣
2
∣ λ − 3 λ + 4 3λ ∣

(1) -5

(2) -4 ✅
(3) -3

(4) -2

Answer: (2)

Solution:
We divide L.H.S. by λ and C by λ 4
1
2
, C2 by λ and C by λ on the R.H.S. to obtain
3

∣ 1 + 3/λ 1 − 1/λ 1 + 3/λ ∣


2 3 4
∣ ∣
1 1 1 1 2
p + q( ) + r( ) + s( ) + t( ) = ∣ 1 + 1/λ 2/λ − 1 1 − 3/λ ∣
λ λ λ λ
∣ ∣
2
∣ 1 − 3/λ 1 + 4/λ 3 ∣

Taking limit as λ → ∞, we get


∣1 1 1∣ ∣1 1 1∣
∣ ∣ ∣ ∣
p = 1 −1 1 = 0 −2 0 = −4
∣ ∣ ∣ ∣
∣1 1 3∣ ∣0 0 2∣

[Applying R 2 → R2 − R1 , R3 → R3 − R1 ]

Q78

If a 1, a2 , a3 … an are in A.P and if 1 + 2x is the arithemetic mean of a 1, an and 1 − x is the arithemetic mean of
2

a1 an
,
1

a2 an−1
,
1

a3 an−2
,…,
1

an a1
and 1 + x is the AM of 2 1

a1
,
1

a2
,
1

a3
,……
1

an
then value of x may be

(1) 1

(2) √2 − 1

(3) √2

www.mathongo.com
AYJR 2024 (January) – Morning Shift Are You JEE Ready (AYJR)


Questions with Answer Keys and Solutions MathonGo
(4) 0

Answer: (4)

Solution:
a1 +an
1 + 2x =
2

a1 + an = a2 + an−1 = a3 + an−2 … .
2 1 1 1
(1 − x ) × n = + + ⋯ +
a1 an a2 an−1 an a1

2 1 1 1
(1 + x ) × n = + + ⋯ +
a1 a2 an

a1 + an a2 + an−1 an + a1 1
2
(1 − x ) × n = [ + + ⋯ + ] ×
a1 an a2 an−1 an a1 (a1 + an )

1 1 1 1 1 1 1
= [ + + + + ⋯ + + ] ×
a1 an a2 an−1 an a1 (a1 + an )

1 1 1 1
2
(1 − x ) × n = 2 [ + + ⋯ + ] ×
a1 a2 an (a1 + an )

1
2 2 2 3 2
(1 − x ) × n = 2 × (1 + x ) × n × ⇒ 1 − x + 2x − 2x = 1 + x
2(1 + 2x)

3 2 2
2x + 2x − 2x = 0 ⇒ 2x (x + x − 1) = 0

−1 ± √5
x = 0,
2

Q79
x

Number of real roots of the equation is -


e
= 5 − x
2
x

(1) 1

(2) 2

(3) 3 ✅
(4) 0

Answer: (3)

Solution:

www.mathongo.com
AYJR 2024 (January) – Morning Shift Are You JEE Ready (AYJR)

Questions with Answer Keys and Solutions MathonGo


x
e
f (x) =
2
x
2 x x
x ⋅ e − 2xe

f (x) =
4
x
x
e (x − 2)
=
3
x

⇒ f (x) = 5 − x has 3 distinct roots.

Q80

2 2 2 2
y y
On a coordinate plane, ellipse C and hyperbola C has the
x x
1
: + = 1 (a1 > b1 > 0) 2
: − = 1 (a2 , b2 > 0)
2 2 2 2
a b a b
1 1 2 2

same focus point F 1, F2 . Point P is the intersection point of C and C in the first quadrant and |F 1 2 1 F2 | = 2 |P F2 | ⋅ e1 is
the eccentricity of C and e is the eccentricity of C . Find the range of e
1 2 2 2 − e1 .

(1) ( 1

3
, ∞)

(2) ( 1

2
, ∞) ✅
(3) [ 1

3
, ∞)

(4) [ 1

2
, ∞)

Answer: (2)

Solution:
By the properties of an ellipse, |P F 1| + |P F2 | = 2a , and by the properties of a hyperbola, |P F 1| − |P F2 | = 2a2 .
These equations combine to |P F 1| = a1 + a2 and |P F 2| = a1 − a2 .
Also by the properties of an ellipse and hyperbola, |F 1 F2 | = 2a1 e1 = 2a2 e2 . Since
|F1 F2 | = 2 |P F2 | , 2a1 e1 = 2 (a1 − a2 ) .
Substituting a and rearranging eliminates a and gives e .
a1 e 1 e1

2
= 1 2
=
e2 1−e1

e1
For a hyperbola, e 2 > 1 , which means 1−e1
> 1 , which solves to e 1 >
1

2
.
For an ellipse, e 1
< 1 Therefore 1

2
< e1 < 1 .
2
e

Therefore, D = e . For , the range of D is ( .


e1 1 1 1
2 − e1 = − e1 = < e1 < 1 , ∞)
1−e1 1−e1 2 2

Q81

3{x}+1
If ∫ , then find the value of n.
n 2
dx = 6 ln(4e )
−n {3x}+1

[Note: {k} denotes the fractional part function of k.]

www.mathongo.com
AYJR 2024 (January) – Morning Shift Are You JEE Ready (AYJR)

Questions with Answer Keys and Solutions MathonGo


Answer: 6

Solution:

1 2
1 1
3{x} + 1 3 3x + 1 3 3x + 1 3x + 1
In = 2n ∫ dx = 2n (∫ dx + ∫ dx + ∫ dx)
0 {3x} + 1 0
3x + 1 1 3x − 1 + 1 2 3x − 2 + 1
3 3

1
1 1 ln 2 2 1 1 ln 2 1 2
1
= 2n ( + + + ∫ (1 + ) dx) = 2n ( + + + + (ln(3x − 1)) 2 )
3 3 3 2 3x − 1 3 3 3 3 3 3
3

ln 2 2 ln 2
2 2
= 2n (1 + + ) = 2n(1 + ln 2) = n ln(4e ) ≡ 6 ln(4e )
3 3

∴ n = 6

Q82

In a village of 1000 inhabitants, there are three newspapers P , Q and R in circulation. Each of these papers is read by
500 persons. Papers P and Q are read by 250 persons, papers Q and R are read by 250 persons, papers R and P are
read by 250 persons. All the three papers are read by 250 persons. Then the number of persons who read no newspaper
at all, is

Answer: 0

Solution:

Let 'a' number of people read no newspaper at all.


Now, x1 + x4 + x5 + x7 = 500

x2 + x5 + x6 + x7 = 500

x3 + x4 + x6 + x7 = 500

x5 + x7 = 250

x4 + x7 = 250

x6 + x7 = 250

x7 = 250

And, x1 + x2 + x3 + x4 + x5 + x6 + x7 + a = 1000

⇒ x4 = x5 = x6 = 0

⇒ x1 = 250, x2 = 250, x3 = 250

⇒ a = 1000 − 250 − 250 − 0 − 0 − 0 − 250 = 0

www.mathongo.com
AYJR 2024 (January) – Morning Shift Are You JEE Ready (AYJR)

Questions with Answer Keys and Solutions MathonGo


Q83

Let f be real function defined on R (the set of real numbers) such that

f (x) = 100(x − 1)(x − 2) (x − 3)
2 3
… . (x − 100)
100
, for all x ∈ R. If g is function defined on R such that
, If sum of all the value of x for which g(x) has a local extremum be λ then find λ
x x
f (t)
∫ e dt = ∫ g(x − t)dt + 2x + 3
a 0

Answer: 2500

Solution:

x x
f (t)
∫ e dt = ∫ g(x − t)dt + 2x + 3
a 0
x x
f (t)
⇒ ∫ e dt = ∫ g(t)dt + 2x + 3
a 0

differential both sides, we get


f (x)
e = g(x) + 2

f (x)
⇒ g(x) = e − 2

′ f (x) ′
⇒ g (x) = e ⋅ f (x)

∵ e
f (x)
is always greater than zero
∴ sign of g ′
(x) is same as sign of f ′
(x)

∵ sign of g ′
(x)

Clearly, local extremum (maximum or minimum) will occur at x = 99, 97, 95, 3, 1
∴ Sum of all the values = 1 + 3 + 5 + … + 99 = 50

2
[2 × 1 + (50 − 1) × 2] = 2500

Q84

a, b, c, d are in GP and are in ascending order such that a + d = 112 and b + c = 48. If the GP is continued with a as
the first term, then the sum of the first six term is

Answer: 1456

Solution:

www.mathongo.com
AYJR 2024 (January) – Morning Shift Are You JEE Ready (AYJR)

Questions with Answer Keys and Solutions MathonGo


a, b, c, d − GP

2 3
a, ar, ar , ar

3
a + ar = 112

2
ar + ar = 48
3
1 + r 112 7
= =
r(1 + r) 48 3

2
1 + r − r 7
=
r 3
2
3r − 3r + 3 = 7r
2
3r − 10r + 3 = 0

r = 3

a(28) = 112

a = 4

6
4 (3 − 1)
s6 = = 2(728)
(3 − 1)

= 1456

Q85

Given A and B are two non singular matrices such that A ≠ I , B 6


= I and BA 3
= AB , then the least value of m for
which A m
= I is K . Then K is (where I is identity matrix) Given (m ∈ N )

Answer: 728

Solution:

3
AB = BA

3
(AB)B = (BA ) B

2
= BA (AB)

2 2 3
AB = BA BA

3
= BA(AB)A

3 3
= BABA A

3 6
= B (BA ) A

2 2 9
AB = B A
n
n n 3
In general AB = B A

put n = 6,
6
6 6 3
AB = B A
6
3 729
A = A = A

728
A = I

K = 728

www.mathongo.com
AYJR 2024 (January) – Morning Shift Are You JEE Ready (AYJR)

Questions with Answer Keys and Solutions MathonGo


Q86

If ā, ¯b are vectors perpendicular to each other & |ā| = 2, |¯b| = 3, and c̄ is a vector such that c̄ ¯
× ā = b then the least
value of 2|c̄ − ā| =

Answer: 3

Solution:

¯
c̄ × ā = b

3
⇒ |c̄ ||ā| sin θ = 3 |c| =
2 sin θ

2
9 3
|c̄ − ā| = + 4 − 2 ⋅ 2 cos θ
2
4 sin θ 2 sin θ

9
2
= 4 + cosec e θ − 6 cot θ
4
2
9 3
= + ( cot θ − 2)
4 2

2
9
|c̄ − ā| ≥
4

⇒ 2|c̄ − ā| ≥ 3

Q87

The remainder when (2024) 2024


is divided by 17 is

Answer: 1

Solution:
Target is to write term in form of 17 K + 1 or 17 K − 1
2024 = 2023 + 1 = 17 × 119 + 1
2024
(2023 + 1) = 2023 K + 1

Hence, Remainder = 1

Q88

The angle between the lines whose direction cosines are given by 3l + 4m + 5n = 0, l 2
+ m
2
− n
2
= 0 is (Mark acute
angle in degree as answer)

Answer: 0

Solution:
2
(3l+4m)
Eliminating n between the relations, l 2
+ m
2
=
25
⇒ 16l
2
− 24lm + 9m
2
= 0

2 3m
(4l − 3m) = 0, l =
4

www.mathongo.com
AYJR 2024 (January) – Morning Shift Are You JEE Ready (AYJR)

Questions with Answer Keys and Solutions MathonGo


∴ The d.r's are (3, 4, −5), (3, 4, −5).
The lines are parallel and the angle between them is zero.

Q89

f : N → R − {0} , such that f (1) + f (2) + … + f (n) = f (n) ⋅ f (n + 1), ∀n ∈ N.


Let f (2) + f (4) + f (6) + … + f (100) = a and s(n) denote the sum of digits of n then s(s(a)) is

Answer: 6

Solution:

f (1) + f (2) + … + f (n) = f (n) ⋅ f (n + 1)∀n ∈ N

For n = 1, f (1) = f (1) ⋅ f (2) ⇒ f (2) = 1,

Similarly f (4) = 2, f (6) = 3 … , f (2n) = n.

50(51)
Hence f (2) + f (4) + … + f (100) = 1 + 2 + … + 50 = = 1275
2

S(S(1275)) = S(15) = 6

Q90

The number of ways of distributing 20 identical marbles among three children so that each gets atleast one and no two
get the same number is

Answer: 144

Solution:
a < b < c, a + b + c = 20

a = 1 ⇒ (b, c) = (2, 17), (3, 16), … (9, 10) : 8 pairs

a = 2 ⇒ (b, c) = (3, 15), (4, 14), … (8, 10) : 6 pairs

a = 3 ⇒ (b, c) = (4, 13), (5, 12), … (8, 9) : 5 pairs

a = 4 ⇒ (b, c) = (5, 11), (6, 10), … (7, 9) : 3 pairs

a = 5 ⇒ (b, c) = (6, 9), (7, 8) : 2 pairs

Total 24

∴ There are 24 triples (a, b, c)


Permuting them, the desired number of ways is 24 × 6 = 144.

www.mathongo.com
AYJR 2024 (January) – Evening Shift Are You JEE Ready (AYJR)

Questions with Answer Keys and Solutions MathonGo

Q1

If force (F ), length (L) and time (T ) be considered fundamental units, then the units of mass will be

(1) [F L T −2
]

(2) [F L −2
T
−1
]

(3) [F L −1
T ]
2

(4) [F 2
L T
−2
]

Answer: (3)

Solution:
Let M ∝[F a b
L T ]
c

Writing dimensions on both sides and using the principle of homogeneity of dimensions we have,
a b c
1 0 0 −2
[M L T ]= k[M L T ] [L] [T]

On comparing the power on both sides we get a = 1, a + b = 0 and −2a + c = 0


On solving we have b = −1, c = 2, a = 1
∴ units of mass is [F L −1
T ]
2

Q2

A symmetrical uniform solid cube of side 5 m is placed on a horizontal surface beside a vertical wall, one side of the
cube is making an angle 45° with the floor as shown. If coefficient of friction μ is the same for both wall and floor, the
minimum value of μ so that cube does not slip

(1) μ = 1

(2) μ = 0 ✅
(3) μ = 1

(4) Impossible to balance for any value of μ

Answer: (2)

www.mathongo.com
AYJR 2024 (January) – Evening Shift Are You JEE Ready (AYJR)

Questions with Answer Keys and Solutions MathonGo


Solution:
At angle 45°, N and Mg balance each other, they both pass through centre of mass, so no need for friction to balance it

Q3

A screw gauge has some zero error, but its value is unknown. We have two identical rods. When the first rod is inserted
into the screw, the state of the instrument is shown by the diagram (I). When both the rods are together in series then
the state is shown by the diagram (II) what is the zero error of the instrument? 1 msd = 100 csd = 1 mm

(1) −0. 16 mm

(2) +0. 16 mm

(3) +0. 14 mm ✅
(4) −0. 14 mm

Answer: (3)

Solution:

12
R1 = 2 + = 2 .12 mm
100

www.mathongo.com
AYJR 2024 (January) – Evening Shift Are You JEE Ready (AYJR)

Questions with Answer Keys and Solutions MathonGo


10
R2 = 4 + = 4.10 mm
100

R1 − Z = l (Z is the zero error)

R2 − Z = 2l (l is the length of rod)

On solving we get,

Z = 2R1 − R2

= 4.24 − 4.10

= + 0.14 mm

Q4

th
A body is projected up with a velocity equal to 3

4
of the escape velocity from the surface of the earth. The height it
reaches from the center of the earth is

(1) 10R/9

(2) 16R/7 ✅
(3) 9R/8

(4) 10R/3

Answer: (2)

Solution:
3
v = ve
4
2
1 2 1 3 9 2 9 2GM
K. E. = mv = m( ve ) = mve = m( )
2 2 4 32 32 R

9 GM m GM m GM m
− = −
16 R R r

16R
r =
7

Q5

A thin prism has different medium on its either side. A light ray is incident almost normally on the first face. What is
the angle of deviation if all the angles are very small

www.mathongo.com
AYJR 2024 (January) – Evening Shift Are You JEE Ready (AYJR)

Questions with Answer Keys and Solutions MathonGo

(1) I (1 −
μ1

μ2
)−A(1 −
μ

μ2
) ✅
(2) I (1 −
μ1 μ
)+A(1 − )
μ2 μ2

μ1 μ
(3) I (1 + μ2
)−A(1 −
μ2
)

(4) none of these

Answer: (1)

Solution:
From Snell's law at first refraction,
μ1 I = μr1
μ1
r1 = I
μ
...(1)
From Snell's law at second refraction,
μr2 = μ2 e

μ(A − r1 )= μ2 e

∴ From (1),
μ1
μ(A − I )= μ2 e
μ

μ μ1
e =
μ2
(A − I
μ
) ...(2)
Overall deviation,
δ = I + e − A

μ μ1
δ = I + (A − I )−A
μ2 μ

μ1 μ
δ = I (1 − )−A(1 − )
μ2 μ2

Q6

The figure shows the acceleration-time graph of a particle. Which of the following represents the corresponding
velocity-time graph? ( consider initial velocity zero )

www.mathongo.com
AYJR 2024 (January) – Evening Shift Are You JEE Ready (AYJR)

Questions with Answer Keys and Solutions MathonGo

(1)

(2)


(3)

(4)

www.mathongo.com
AYJR 2024 (January) – Evening Shift Are You JEE Ready (AYJR)

Questions with Answer Keys and Solutions MathonGo


Answer: (2)

Solution:
Since acceleration is constant, therefore there is uniform increase in velocity. So, the v − t graph is a straight line
slopping upward to the right. When acceleration becomes zero, velocity is constant. So v − t graph is a straight line
parallel to the time-axis.

Q7

A 100% efficient transformer has 100 turns in the primary and 25 turns in its secondary coil. If the current in the
secondary coil is 4 A, then the current in the primary coil is,

(1) 2 A

(2) 3 A

(3) 1.3 A

(4) 1 A ✅
Answer: (4)

Solution:

VP NP
In a transformer, we have V
=
N
, where V is the voltage and N is the number of turns in coil. P and S represents
S S

the primary and secondary coils respectively.

According to question, N P = 100 ,N S = 25 , IS = 4 A .

VP 100
=
VS 25

Power remains same in input and output windings in 100% efficient transformer, ⇒ V P IP = VS I S .
VS
IP =
VP
IS =
1

4
× 4 = 1 A .

Q8

An ideal gas expands isothermally from volume V to V and then it is adiabatically compressed back to its original
1 2

volume V . The initial and final pressures of the gas are P and P respectively and the net work done by the gas is W
1 1 3

, then

(1) P 3 > P1 , W > 0

www.mathongo.com
AYJR 2024 (January) – Evening Shift Are You JEE Ready (AYJR)

Questions with Answer Keys and Solutions MathonGo


(2) P 3 < P1 , W < 0

(3) P 3 > P1 , W < 0 ✅


(4) P 3
= P1 , W = 0

Answer: (3)

Solution:
The two processes are shown in the following P-V diagram:

For iothermal process:


P1V1 = P2V2
V2
i.e., P1 =(
V1
)P2

For adiabatic process:


γ γ
P3 V = P2 V
1 2
γ
V2
i.e. P3 = (
V1
) P2

As γ > 1 , hence P3 > P1


Further, as slope of adiabatic curve is greater than that of isothermal process curve, adiabatic curve will lie above the
isothermal curve. That is, area under adiabatic curve > area under isothermal curve
i.e., Negative work > Positive work
i.e., W < 0

Q9

Two ends of a conducting rod of varying cross-sections are maintained at 200°C and 0°C respectively. In steady-state

www.mathongo.com
AYJR 2024 (January) – Evening Shift Are You JEE Ready (AYJR)

Questions with Answer Keys and Solutions MathonGo


(1) temperature difference across AB and CD are equal

(2) temperature difference across AB is greater than that of across CD

(3)

temperature difference across AB is less than that of across CD


(4) temperature difference may be equal or different depending on the thermal conductivity of the rod

Answer: (3)

Solution:
dQ
Rate of flow of heat dt
or H is equal throughout the rod.
Temperature difference = (H) X (thermal resistance)
or Temperature difference (μ) ∝ thermal resistance (R)
where R =
l

KA
or R ∝
A
1

Area across CD is less. Therefore, temperature difference across CD will be more.

Q10

Assertion: Each bulb in a frill of 20 bulbs in series, when connected to supply voltage, will emit more light than each
bulb in a frill of 19 bulbs in series when connected to same supply voltage.
Reason: Each bulb in a frill of 20 bulbs in series will get more voltage than that in a frill of 19 bulbs.

(1) If both Assertion and Reason are true and Reason is correct explanation of Assertion.

(2) If both Assertion and Reason are true but Reason is not explanation of Assertion.

(3) If Assertion is true but Reason is false.

(4) If Assertion is false and Reason is false. ✅


Answer: (4)

Solution:
Power consumption in an electric circuit for a particular supply voltage is,
2
V
P =
Req

1
⇒ P ∝
Req

So, the circuit with higher resistance will consume lesser power and hence, for the given values,

www.mathongo.com
AYJR 2024 (January) – Evening Shift Are You JEE Ready (AYJR)

Questions with Answer Keys and Solutions MathonGo


R20 = 20R & R19 = 19R

⇒ R20 > R19

⇒ P20 < P19

For 20 bulbs, the source voltage is divided to 20 whereas, for 19 bulbs, it is divided to 19. So, each bulb gets more
voltage. Thus, Assertion is false and Reason is false.

Q11

Choose the wrong statement

(1) the radius of path of a charged particle moving in a uniform magnetic field is proportional to the momentum of the
particle

(2) an electron beam is moving towards east, on which a perpendicular magnetic field is acting upwards. The beam will
be deflected towards the north direction

(3) a positive charge is going straight away from the observer. The magnetic line of force produced due to it are in
clockwise direction.


(4) while passing through a given place, the path of electron remains straight line. It can be definitely said that the
magnetic field is not present at that place

Answer: (4)

Solution:
If the path of electron remains straight line then either magnetic field is not present at that place or velocity of electron
is parallel to the magnetic field.

Q12

A magnet is cut in four equal parts by cutting it parallel to its length. The time period of each part, if the time period of
the original magnet in the same field is T , will be 0

(1) T 0 ✅
T0
(2) 2

T0
(3) 4

(4) 4T 0

Answer: (1)

Solution:
For each part, I 1
=
I

4
and M 1
=
M

4
.
www.mathongo.com
AYJR 2024 (January) – Evening Shift Are You JEE Ready (AYJR)

Questions with Answer Keys and Solutions MathonGo


I1
∴ T = 2π√(
MB
I
) and T 1 = 2π√(
M1 B
)

I×4
= 2π√( )
4M×B

T1
∴ = 1
T

∴ T1 = T = T0

Q13

Imagine a light planet revolving around a very massive star in a circular orbit of radius R with a period of revolution T
5

. If the gravitational force of attraction between the planet and the star is proportional to R −
2
, then

(1) T is proportional to R
2 2

(2) T
2
is proportional to R
7

2

3

(3) T
2
is proportional to R 2

(4) T
2
is proportional to R 2

Answer: (2)

Solution:
5

Given, F = K R

2

We know, F = mω R
2

7
2 −
⇒ ω ∝ R 2

⇒ T
2
∝ R2 ( since T ∝ 1

ω
)

Q14

Match the List - I (Phenomenon associated with electromagnetic radiation) with List - II (Part of electromagnetic
spectrum) and select the correct code from the choices given below the lists :

List - I List-II
Visible
I Doublet of sodium A
radiation
Wavelength corresponding to temperature associated with
II B Microwave
the isotropic radiation filling all space
Short
III Wavelength emitted by atomic hydrogen in interstellar space C
radiowave

www.mathongo.com
AYJR 2024 (January) – Evening Shift Are You JEE Ready (AYJR)

Questions with Answer Keys and Solutions MathonGo


List - I List-II
Wavelength of radiation arising from two close energy
IV D X-rays
levels in hydrogen

(1) (I)-(D), (II)-(C), (III)-(A), (IV)-(B)

(2) (I)-(A), (II)-(B), (III)-(C), (IV)-(C) ✅


(3) (I)-(A), (II)-(B), (III)-(B), (IV)-(C)

(4) (I)-(B), (II)-(A), (III)-(D), (IV)-(A)

Answer: (2)

Solution:
(I)-(A), (II)-(B), (III)-(C), (IV)-(C)

Q15

A clear sheet of Polaroid is placed on top of a similar sheet so that their axes make an angle of sin
−1
(
4

5
) with each
other. The ratio of the intensity of the emergent light to that of polarised light is

(1) 16 : 25

(2) 9 : 25 ✅
(3) 4 : 5

(4) 8 : 25

Answer: (2)

Solution:

Given,

angle between polariser and analyser, θ = sin −1


(
4

5
)

4
⇒ sinθ =
5

2 3
∴ cosθ = √1 − sin θ =
5

Intensity of transmitted light is given by law of Malus,

2
I = I0 cos θ

where I and I are the intensities of the polarised and emergent light respectively.
0

www.mathongo.com
AYJR 2024 (January) – Evening Shift Are You JEE Ready (AYJR)

Questions with Answer Keys and Solutions MathonGo


I 2
∴ = cos θ
I0

2
I 3
⇒ = ( )
I0 5

I 9
⇒ =
I0 25

Hence, the required ratio is 9 : 25.

Q16

The graph between maximum kinetic energy and intensity of light in photoelectric effect is plotted. Out of the four
graphs A, B, C, D shown in the figure, the correct graph is

(1) C

(2) B

(3) D ✅
(4) A

Answer: (3)

Solution:
The maximum kinetic energy of the emitted photoelectrons depends on the frequency of the incident light but is
independent of the intensity of the incident light.

Q17

Consider the nuclear fission reaction w → x + y, now using the graph given answer the following question.

www.mathongo.com
AYJR 2024 (January) – Evening Shift Are You JEE Ready (AYJR)

Questions with Answer Keys and Solutions MathonGo

What is the Q-Value of the reaction?

(1) E 1
N1 −[E2 N2 + E3 N3 ]

(2) [E 2 N2 + E3 N3 ]−E1 N1 ✅
(3) [E 2 N2 + E1 N1 ]−E3 N3

(4) [E 1
N1 + E3 N3 ]−E2 N2

Answer: (2)

Solution:
For energy release the total binding energy of product should be more than that of reaction.

Q18

The 6 V Zener diode shown in the figure has negligible resistance and a knee current of 5 mA . The minimum value of
R (in Ω) so that the voltage across it does not fall below 6 V is

(1) 40

(2) 60

(3) 72

www.mathongo.com
AYJR 2024 (January) – Evening Shift Are You JEE Ready (AYJR)


Questions with Answer Keys and Solutions MathonGo
(4) 80

Answer: (4)

Solution:

In Zener diode,
Vmax −VZ
⇒ ≥ Iknee + IR
R1

R1 is the series resistance of 50 Ω and I is the current through the resistance R


R

10−6
⇒ ≥ 5m + IR
50

⇒ IR ≤ 80 mA − 5 mA

∵ (IR )
max
= 75 mA (the maximum current through R)

So the minimum value of the resistance R is


V 6 V 6000
Rmin = = = = 80 Ω
( IR ) 75 mA 75
max

Q19

Two equally charged small metal balls placed at a fixed distance experience a force F . A similar uncharged metal ball
after touching one of them is placed at the middle point between the two balls. The force experienced by this ball is

(1) F


2

(2) F

(3) 2F

(4) 4F

Answer: (2)

Solution:
2
q.q q
Initially, F AB
=
4πε0
1
.
2
=
1

4πε0
.
2
r r

Finally, force on C

www.mathongo.com
AYJR 2024 (January) – Evening Shift Are You JEE Ready (AYJR)

Questions with Answer Keys and Solutions MathonGo


q q q
( ) (q) ( ) ( ) 2
1 2 1 2 2 1 q
FC = FCB − FCA = . − . = .
4πε0 2 4πε0 2 4πε0 2
r r r
( ) ( )
2 2

So F C = FAB = F

Q20

A closed and an open organ pipe have the same length. When they are vibrating simultaneously in their first overtone,
they produce three beats. The length of the open pipe is now made one third the original length and one of its ends is
closed. On the other hand, the length of the closed pipe is made three times the original length. The number of beats
produced when they vibrate with fundamental frequencies will be

(1) 8 ✅
(2) 14

(3) 17

(4) 12

Answer: (1)

Solution:
V = speed of sound and L = lenght of pipes
For open pipe first overtone f
V
1 =
L

For closed pipe first overtone f '


3V
1 =
4L
V 3V
∴ f 1 − f '1 = − = 3
L 4L
V
∴ = 3
4L
V
∴ = 12
L

When we close one end of the open pipe and make its length one-third of the original length, then its fundamental
frequency is
V 3V
f = =
L
4L
4( )
3

When the length of closed pipe is made 3 times, the fundamental frequency is
V V
f' = =
4(3L) 12L

Beats produced = f − f '


3V V
= −
4L 12L

8 V 8
= . = × 12
12 L 12

= 8

Q21

www.mathongo.com
AYJR 2024 (January) – Evening Shift Are You JEE Ready (AYJR)

Questions with Answer Keys and Solutions MathonGo


Two wires having the same length and material are stretched by the same force. Their diameters are in the ratio 1 : 3.
The ratio of strain energy per unit volume for these two wires (smaller to larger diameter), when stretched, is M:1, find
M.

Answer: 81

Solution:
Strain energy per unit volume u = U
=
1

2
× stress × strain
V
U 1 F Δl
= × ×
V 2 A l
U 1 Stress
= Stress ×
V 2 Y
2 2
uS YL Strain(S) AL
∴ = × ( ) = ( )
uL YS Stress(S) AS

2
2
r
L 81
= ( ) = ( )
2 1
r
S

Q22

Water from a tap emerges vertically downwards with an initial speed of 1. 0 m s . The cross-sectional area of the tap −1

is 10 −4
m
2
. If we assume that the pressure is constant throughout and that the flow is steady, then the cross-sectional
area of the stream, 0. 15 m below the tap, is n × 10 −5
m
2
. What is the value of n?

Answer: 5

Solution:
From conservation of energy
v
2
2
= v
2
1
+ 2gh ...(i)
[can also be found by applying Bernouilli's theorem between 1 and 2]
From continuity equation
A1 v 1 = A2 v 2

A1
v2 =(
A2
)v1 ...(ii)
Substituting value of v2 from Eq. (ii) in Eq. (i)

www.mathongo.com
AYJR 2024 (January) – Evening Shift Are You JEE Ready (AYJR)

Questions with Answer Keys and Solutions MathonGo


2
A
1 2 2
⋅ v = v + 2gh
2 1 1
A
2
2 2
A v

or
2 1 1
A =
2 2
v +2gh
1

A 1 v1
∴ A2 =
2
√v +2gh
1

Substituting the given values


−4 2
( 10 m ) ( 1.0 m/s )

A2 =
2
√ ( 1.0 m/s ) +2 ( 10 ) ( 0.15 )

−5 2
A2 = 5 × 10 m

Q23

An object and a concave mirror are approaching each other with velocities 10 m/s and 5 m/s as shown in figure. The
velocity of image of object at the instant shown in figure is V, find absolute value of V.

Answer: 65

Solution:
The formula for velocity of image is given by :
f −10
= VIm = = = −2
f −u −10+15

∴ VIm = −4 VOm

Therefore, ˆ ˆ ˆ
V I − 5(− i )= −4{10 i + 5 i }


ˆ
⇒ V I = −65 i

Q24

A flexible circular loop 20 cm in diameter lies in a magnetic field of magnitude B = 1 T , directed into the plane of
page as shown. The loop is pulled at the points indicated by the arrows forming a loop of zero area in 0. 314 sec . The
average emf induced (in V) in the loop is K, find 10K. (π = 3.14)

www.mathongo.com
AYJR 2024 (January) – Evening Shift Are You JEE Ready (AYJR)

Questions with Answer Keys and Solutions MathonGo

Answer: 1

Solution:

Δϕ
The formula of induced emf is equal to the rate of change of magnetic flux, ⇒ emf = Δt

and we know that magnetic flux is the scalar product of the magnitude of magnetic field and cross-sectional area,
2
π ( 0.1 ) ×B
emf = = 0. 1 V
0.314

K = 0.1 V

10 K = 1

Q25

In the circuit shown in the figure, calculate the charge on 2 μF capacitor in steady-state (in μC ).

Answer: 40

Solution:

www.mathongo.com
AYJR 2024 (January) – Evening Shift Are You JEE Ready (AYJR)

Questions with Answer Keys and Solutions MathonGo

Given,

Capacitance of Capacitor C 1
= 2 μF .

Capacitance of Capacitor C 2 = 4 μF

EMF of batteries are E 1 = 20 V ,E 2 = 10 V .

As the same current flows through C and C . 1 3

Therefore, they are in series and have same charge q . 1

Applying KVL in the loops,


q2 q2
⇒ 10 − + 20 − = 0
2 4

3q2
⇒ 30 − = 0
4

⇒ q2 = 40 μF

Q26

6√3
A car is moving in a circular horizontal track of radius 5
m . A plumb bob is suspended from the roof of the car by a
light rigid rod. The angle made by the rod with the vertical is 60 . Then the car moves with a constant speed of
o

_______ m/s.(g = 10 m/s 2


)

Answer: 6

Solution:

www.mathongo.com
AYJR 2024 (January) – Evening Shift Are You JEE Ready (AYJR)

Questions with Answer Keys and Solutions MathonGo

2
mv
T sin θ =
R

T cos θ = mg
2
v
∴ tan θ =
Rg

2
∴ v = tan θ × R × g

o 6√3
2
∴ v = tan 60 × × 10
5

6√3
2
∴ v = √3 × × 10
5

2
∴ v = 36

∴ v = 6 m/s

Q27

A stationary H e ion emitted a photon corresponding to the first line of the Lyman series. The photon liberates electron
+

from a stationary hydrogen atom in the ground state. The velocity of the liberated electron is 3.1 × 10 x
m/s . Find x
(You can make necessary approximations)

Answer: 6

Solution:
Energy of a photon emitted for transition n to n is given by 2 1

1 1
ΔE = 13.6 × 4 ( − ) eV … (1)
2 2
n n
1 2

1 1
ΔE = 13.6 × 4 ( − ) eV = 40.8eV … (2)
2 2
1 2

1
2
mv = (40.8 − 13.6)eV = 37.2eV
2
6
∴ v = 3.1 × 10 m/s

∴ x = 6

Q28

www.mathongo.com
AYJR 2024 (January) – Evening Shift Are You JEE Ready (AYJR)

Questions with Answer Keys and Solutions MathonGo


Two wires A and B of the same material, have radii in the ratio 1 : 2 and carry currents in the ratio 4 : 1. The ratio of
drift speed of electrons in A and B is K:1, find K.

Answer: 16

Solution:
Drift speed of electrons is given by
I = neAvd

Therefore v d

I

2
r
2
(vd )A IA r
B
= ×
(v )B IB 2
d r
A

(vd )A 2
4 2 16
= × =
(vd )B 1 2 1
1

Q29

A disc of mass m and radius R is attached to a rectangular plate of the same mass, breadth R and length 2R as shown
in figure. The moment of inertia of the system about the axis AB passing through the centre of the disc and along the
plane is I =
1

α
(
31

3
mR )
2
, where value of α is

Answer: 4

Solution:
2

IAB = Idisc + Iplate + M (


3R

2
) (From Parallel Axis Theorem)
2
2
1 2 MR 3R
= MR + + M( )
4 12 2

31 2
= MR
12

Q30

The relationship between the force F and position x of a body is as shown in the figure. The work done in displacing
the body from x = 1 m to x = 5 m will be-

www.mathongo.com
AYJR 2024 (January) – Evening Shift Are You JEE Ready (AYJR)

Questions with Answer Keys and Solutions MathonGo

Answer: 15

Solution:
In F − x graph, the total area under the curve gives total work done. To calculate the total area.
Area = 10 + 5 − 5 + 5 = 15 J

Q31

End product of the following sequence of reactions is:

(1)

(2)


(3)

www.mathongo.com
AYJR 2024 (January) – Evening Shift Are You JEE Ready (AYJR)

Questions with Answer Keys and Solutions MathonGo


(4)

Answer: (2)

Solution:
The first step is an allylic bromination of cycloalkene.(free radical substitution)

NBS is a Br2 source: Br − Br → 2Br

Q32

AgNO3 (aq) was added to an aqueous KCl solution gradually and the conductivity of the solution was measured. The
plot of conductivity (Λ) versus the volume of AgNO is 3

(1) (P)

(2) (Q)

(3) (R)

(4) (S) ✅
Answer: (4)

Solution:
AgNO3 + KCl(aq)→ AgCl(s)+KNO3 (aq)

Initially there is aq. KCl solution now as solution of AgNO is added, AgCl(s) is formed. Hence conductivity of
3

www.mathongo.com
AYJR 2024 (January) – Evening Shift Are You JEE Ready (AYJR)

Questions with Answer Keys and Solutions MathonGo


solution is almost compensated (or slightly increase) by the formation of KNO . After end point conductivity
3

increases more rapidly because addition of excess AgNO solution.


3

Q33

The chloride of a metal contains 71% chlorine by weight and the vapour density of it is 50. The atomic weigh of the
metal will be:

(1) 29 ✅
(2) 58

(3) 35.5

(4) 71

Answer: (1)

Solution:
Mol. wt. of metal chloride = 50 × 2 = 100
Let metal chloride be metal chlorine then
Wm WCl
Equivalent of metal = Equivalents of chlorine =
29 71
= or =
Em E E 35.5
Cl

29
E =
2

Now a + 35.5n = 100


or n × E + 35 × 5n = 100

∴ n = 2

Therefore, a = 2 × E = 2 ×( 29

2
)= 29

Q34

+
H3 O Δ

The final product in the following reaction is: +KCN → A −−→ B → C

(1)

(2) R − CH 2 − COOH ✅

www.mathongo.com
AYJR 2024 (January) – Evening Shift Are You JEE Ready (AYJR)

Questions with Answer Keys and Solutions MathonGo

(3)

(4) A and B both

Answer: (2)

Solution:

This reaction is used to differentiate between geminal and vicinal dihalide. A geminal dihalide gives acid, whereas
vicinal dihalide gives anhydride if treated with KCN , followed with hydrolysis & heating the product formed. − CN

group on acid hydrolysis always converts to − COOH . Secondly, two− COOH groups on vicinal carbon atom, on
heating always lose water.

Q35

Match the methods in column I with the respective intermediate in column II.

Column I Column II
(i) Dumas method (a) Ammonium sulphate
(ii) Kjeldahl's method (b) Silica gel
(iii) Carius method (c) AgNO
3

(iv) Chromatography (d) Nitrogen gas

(1) i − b, ii − c, iii − a, iv − d

(2) i − a, ii − b, iii − c, iv − d

(3) i − b, ii − a, iii − c, iv − d

(4) i − d, ii − a, iii − c, iv − b ✅
Answer: (4)

Solution:

www.mathongo.com
AYJR 2024 (January) – Evening Shift Are You JEE Ready (AYJR)

Questions with Answer Keys and Solutions MathonGo


There are two methods for the estimation of nitrogen: Dumas and Kjeldahl's method.
In Dumas method, nitrogen gas is evolved which is not absorbed by the solution and by measuring the volume of it, the
percentage of nitrogen is estimated.

In Kjeldahl's method, the organic compounds containing nitrogen convert into ammonium sulphate when treated with
concentrated sulphuric acid.
Δ

O. C. +H2 SO4 → (NH4 ) SO4


2

Δ
2 NaOH → Na2 SO4 + NH3 + 2H2 O

2 NH3 + H2 SO4 → (NH4 ) SO4


2

Kjeldahl method is not applicable to the compounds containing nitrogen in nitro and azo groups and nitrogen present in
the ring as nitrogen of these compounds does not change to ammonium sulphate under these conditions.

Carius method is used for the estimation of halogens, where, silver nitrate is used to react with the halide salts to form
silver halide. The percentage of halogens can be calculated by this silver halide (AgX).

Chromatography is a technique for the separation of a mixture of liquids, where, silica gel is used as an adsorbent
which absorbs the basic contents of the mixture that has to be separated.

Q36

In the given reaction


Red P+Br2 NH3

R − CH2 − COOH −−−−−−→ A−→ Alanine

The name of I reaction and group R is -


st

(1) Hell Vohlard Zelinsky reaction, - C 6 H5

(2) Hell Vohlard Zelinsky reaction, - CH 3 ✅


(3) Schimdt Reaction,

(4) Claisen Schimdt reaction, - CH 3

Answer: (2)

Solution:
Alanine is

and product in given reaction is

www.mathongo.com
AYJR 2024 (January) – Evening Shift Are You JEE Ready (AYJR)

Questions with Answer Keys and Solutions MathonGo


A → R − CH − COOH

Br

So R = −CH 3

I
st
reaction is Hell Vohlard Zelinsky reaction.

Q37

Find the correct match for the column I , from the column II and choose the correct option:
Column - I Column - II
(a) C2 H2 (i) sp d
3 2
hybridisation
(b) SF6 (ii) sp d
3 3
hybridisation
(c) SO2 (iii) sp hybridisation
(d) IF7 (iv) sp
2
hybridisation

(1) (a)⟶(i), (b)⟶(iii), (c)⟶(ii), (d)⟶ (iv)

(2) (a)⟶(iii), (b)⟶(i), (c)⟶(iv), (d)⟶(ii) ✅


(3) (a)⟶(ii), (b)⟶(iii), (c)⟶(i), (d) ⟶(iv)

(4) (a)⟶(iv), (b)⟶(i), (c)⟶(iii), (d)⟶(ii)

Answer: (2)

Solution:
In C 2, H2 , C undergoes, sp hybridisation.
Ground state ↑↓ ↑ ↑

2s 2p

Excited state ↑ ↑ ↑ ↑ H − C ≡ C − H

sp

In SF 6, S undergoes sp 3
d
2
hybridisation.
Ground state ↑↓ ↑↓ ↑ ↑

3s 3p 3d

Excited state ↑ ↑ ↑ ↑ ↑ ↑


3 2
sp d

www.mathongo.com
AYJR 2024 (January) – Evening Shift Are You JEE Ready (AYJR)

Questions with Answer Keys and Solutions MathonGo

In SO 2, S undergoes sp hybridisation
2

In IF 7, I undergoes sp 3
d
3
hybridisation.
Ground state ↑↓ ↑↓ ↑↓ ↑

3s 3p 3d

Excited state ↑ ↑ ↑ ↑ ↑ ↑ ↑

3 2
sp d

Q38

Which of the following compound will undergo tautomerism?

(1)

(2)

www.mathongo.com
AYJR 2024 (January) – Evening Shift Are You JEE Ready (AYJR)

Questions with Answer Keys and Solutions MathonGo


(3)


(4)

Answer: (3)

Solution:
This compound has α -H on saturated α -carbon atom so it can show tautomerism.

In rest other compounds there is no α -H-atom on saturated α -Carbon so no tautomerism is possible in them.

Q39

Amongst the halides, what is the correct order of decreasing Lewis acid character?
(1) BCl 3

(2) AlCl 3

(3) GaCl 3

(4) InCl 3

(1) 1 > 2 > 3 > 4 ✅


(2) 4 > 3 > 2 > 1

(3) 3 > 4 > 2 > 1

(4) 2 > 3 > 4 > 1

Answer: (1)

www.mathongo.com
AYJR 2024 (January) – Evening Shift Are You JEE Ready (AYJR)

Questions with Answer Keys and Solutions MathonGo


Solution:
Lewis acid strength of group 13 halides follow the order: BCl 3 > AlCl3 > GaCl3 > InCl3 .
As we move down the group, the size of atom increases, and as a result, the tendency to attract electrons decreases.
This leads to a decrease in Lewis acid nature down the group.

Q40

A heating coil is immersed in a 100g sample of H 2 O(l) at 1 atm and 100 o


C in a closed vessel.In this heating process,
60% of the liquid is converted to the gaseous form at constant pressure of 1 atm. The densities of liquid and gaseous
water under these conditions are 1000 kg/m and 0.60 kg/m respectively. Magnitude of the work done for the
3 3

process is: (take 1 atm = 10


5
N
/m2 )

(1) 4997 J

(2) 4970 J

(3) 9994 J ✅
(4) 1060 J

Answer: (3)

Solution:
w = −Pext (Vf − Vi )
−3 −3 −3
5 60×10 40×10 100×10
= −10 ( + − )
0.60 1000 1000

5 −3 −3 −3
= −10 (100 × 10 + 0.04 × 10 − 0.1 × 10 ) ;|w|= 9994 J

Q41

Consider the reaction

The alkene formed in major amount is

(1)

www.mathongo.com
AYJR 2024 (January) – Evening Shift Are You JEE Ready (AYJR)

Questions with Answer Keys and Solutions MathonGo


(2)


(3)

(4)

Answer: (2)

Solution:
It is dehydration of alcohol by E mechanism through carbocation intermediates. Carbocation formed undergoes
1

rearrangement and produces more substituted alkene as major product.

www.mathongo.com
AYJR 2024 (January) – Evening Shift Are You JEE Ready (AYJR)

Questions with Answer Keys and Solutions MathonGo

Q42

Which of the following orders of ionic radii is correctly represented?

(1) H −
> H > H
+

(2) Al 3+
> Mg
2+
> N
3–

(3) F –
> O
2–
> Na
+

(4) Na +
> F

> O
2−

Answer: (1)

Solution:
As size ∝ ratio for ions.
e

Size of anion > size of atom > size of cation


− +
H > H > H

In case of F −
,O
−2
, Na
+

radii order is O −2
> F

> Na
+

correct order for Al +3


, Mg
+2
,N
−3
is N −3
> Mg
+
> Al
+3

Q43

Solid Na 2 SO4 is slowly added to a solution which is 0.020 M in Ba(NO 3)


2
and 0.020 M in Pb(NO 3)
2
. Assume that
there is no increase in volume on adding Na 2 SO4 . There preferential precipitation takes place. What is the
concentration of Ba 2+
when PbSO starts to precipitate?
4

[K
sp
(BaSO4 )= 1.0 × 10
−10
and K sp (PbSO4 )= 1.6 × 10
−8
]

(1) 5.0 × 10-9 M

(2) 8.0 × 10-7 M

(3) 1.25 × 10-4M ✅


(4) 1.95 × 10-8M

Answer: (3)

Solution:
When BaSO begins to precipitate
4

Ksp [ BaSO4 ] −10


2− 1.0×10
[SO ]= =
4 2+ 0.020
[ Ba ]

−9
= 5.0 × 10 M

When PbSO begins to precipitate


4

www.mathongo.com
AYJR 2024 (January) – Evening Shift Are You JEE Ready (AYJR)

Questions with Answer Keys and Solutions MathonGo


Ksp ( PbSO4 ) −8
2− 1.6×10
[SO ]= =
4 2+
[ Pb ] 0.020

−7
= 8.0 × 10 M

[SO
2−

4
] is less for BaSO precipitation so BaSO precipitates first when PbSO begins to precipitate
4 4 4

Ksp ( BaSO4 )

[SO
2−

4
]= 8.0 × 10
−7
M at that point concentration of Ba 2+
=
2−
[ SO ]
4

−10
1.0×10 −4
= = 1.25 × 10 M
−7
8.0×10

Q44

At 35 o
C the vapour pressure of CS is 512 mm Hg, and of acetone, CH
2 3 COCH3 , is 344 mm Hg. A solution of CS 2

and acetone in which the mole fraction of CS is 0.25 has a total vapour pressure of 600 mm Hg. Which of the
2

following statements about solution of acetone - CS is true ? 2

(1) A mixture of 100 ml of acetone and 100 ml of CS has a total volume of 200 ml. 2

(2) When acetone and CS are mixed at 35 2


o
C , heat must be absorbed in order to produce a solution at 35 o
C . ✅
(3) When acetone and CS are mixed at 35 2
o
C , heat is released.

(4) Raoult's law is obeyed by both CS and acetone for the solution in which the mole fraction of CS is 0.25.
2 2

Answer: (2)

Solution:
PT = PA °XA + PB °XB = 512 × 0 .25 +(344 × 0 .75)= 386

∴ The solution is showing positive deviation


So For a solution of
Acetone + CS2 ; ΔH = + ve

Q45

STATEMENT-1: The rate of a chemical reaction increases with increase in temperature.


STATEMENT-2: Increase in temperature increases the number of effective collision.

(1) STATEMENT-1 is True, STATEMENT-2 is True, STATEMENT-2 is correct explanation of STATEMENT-1 ✅


(2) STATEMENT-1 is True, STATEMENT-2 is True, STATEMENT-2 is NOT correct explanation of STATEMENT-1

(3) STATEMENT-1 is True, STATEMENT-2 is False

(4) STATEMENT-1 is False, STATEMENT-2 is True

Answer: (1)

www.mathongo.com
AYJR 2024 (January) – Evening Shift Are You JEE Ready (AYJR)

Questions with Answer Keys and Solutions MathonGo


Solution:
On increasing temperature number of effective collision increases which increases rate of chemical reaction.

Q46

Which of the following is the intermediate in the reduction of nitrobenzene

(1) C 6
H5 N = O ✅
(2) C 6 H5 NH − NH − C6 H5

(3) C 6 H5 − N = N − C 6 H5

(4)

Answer: (1)

Solution:

Q47

Which of the following can give iodoform test ?


O O
|| ||

(I) CH3 − C − CH2 − C − CH3

O
||

(II) C6 H5 − CH2 − C − CH3

www.mathongo.com
AYJR 2024 (January) – Evening Shift Are You JEE Ready (AYJR)

Questions with Answer Keys and Solutions MathonGo


(III) CH3 − CHO

O
||

(IV) C6 H5 − C − CH3

(1) Only IV

(2) II and IV

(3) III and IV

(4) All of these ✅


Answer: (4)

Solution:

Anything that has group gives positive iodoform

Q48

It is said that coordination compounds have great importance in biological systems. In this context, which of the
following statements is incorrect?

(1) Chlorophylls are green pigments in plants and contain calcium. ✅


(2) Cyanocobalamine is B and contains cobalt.
12

(3) Carboxypeptidase-A is an enzyme and contains zinc.

(4) Haemoglobin is the red pigment of blood and contains iron.

Answer: (1)

Solution:

Chlorophyll are the green pigments in plants containing magnesium, and not calcium. The chemical formula for
chlorophyll is C55 H72 O5 N4 Mg . It is the magnesium in chlorophyll which is responsible for carrying out the process
of photosynthesis by capturing the sunlight.

Cyanocobalamine is the other name for the vitamin complex B , having the chemical formula C
12 63 H88 CoN14 O14 P .
Therefore, it is a complex of cobalt.

Carboxypeptidase -A is a metalloenzyme of zinc produced in the pancreas of human beings.

www.mathongo.com
AYJR 2024 (January) – Evening Shift Are You JEE Ready (AYJR)

Questions with Answer Keys and Solutions MathonGo


Haemoglobin is the red pigment present in blood, used to transport oxygen from lungs to the tissues. The red colour of
the pigment is due to the presence of iron in it. Therefore, haemoglobin is a complex of iron.

Q49

When MnO is fused with KOH and KNO , a coloured compound is formed. Choose the right compound with the
2 3

appropriate colour.

(1) K 2 MnO4 , green ✅


(2) KMnO , purple
4

(3) Mn 2 O3 , brown

(4) Mn 3 O4 , black

Answer: (1)

Solution:
When MnO reacts with fused mixture of KOH and KNO , it forms K
2 3 2 MnO4 (Green compound) as follows:
Δ
MnO2 + KNO3 + 2 KOH → K2 MnO4 + KNO2 + H2 O
( green )

Q50

If one strand of DNA has a nucleotide sequence 3' ATTCGCTAT 5', the nucleotide sequence of other DNA strand will
be

(1) 3’ TAAGCGATA 5’

(2) 5’ TAGCACGTA 5’

(3) 5’ TAGCACGTA 3’

(4) 5’ TAAGCGATA 3’ ✅
Answer: (4)

Solution:
In a double-stranded DNA, the sequence of nucleotides is complementary to each other, i.e., A pairs with T and G pairs
with C. Hence, the sequence of nucleotide for 3' ATTCGCTAT 5' will be 5' TAAGCGATA 3'.

www.mathongo.com
AYJR 2024 (January) – Evening Shift Are You JEE Ready (AYJR)

Questions with Answer Keys and Solutions MathonGo


Q51

Consider the following cell reaction


+ 2+
2Fe(s)+O2 (g)+4H (aq)→ 2Fe (aq)+2H2 O(l)

If E cell
= E
o

cell
at 25 o
C and [Fe 2+
]= 10
−3
M, PO = 0.01
2
atm and pH = x
Value of x is

Answer: 1

Solution:
For E cell = E
o
cell

log Q must be zero


Q = 1
2+ 2
[ Fe ] −6
10
Q = ⇒ = 1
+ 4 − 4x
[H ] [ PO ] (10 ) ( 0.01 )
2

−4 −4x
10 = 10

x = 1

Q52

If molar mass of compound B is x then find x

Answer: 99

Solution:

Molar mass of B = 198 = x

www.mathongo.com
AYJR 2024 (January) – Evening Shift Are You JEE Ready (AYJR)

Questions with Answer Keys and Solutions MathonGo


x 198
= = 99.00
2 2

Q53

The electrophile involved in above reaction has ______ lone pair of electrons on central carbon atom.

Answer: 1

Solution:

The electrophile

E is singlet carbene so it has on lone pair.

Q54

www.mathongo.com
AYJR 2024 (January) – Evening Shift Are You JEE Ready (AYJR)

Questions with Answer Keys and Solutions MathonGo


How many acidic H-atoms are present in this compound that can react with R for R-MgX to give alkane

Answer: 6

Solution:

Here 6 H-atoms are acidic enough to react with R for R-MgX.


−CH − NO2

Here hydrogen atom of −SO 3 H, −COOH, −C ≡ CH, −OH (phenolic and alcoholic both) and | are
NO2

acidic.

Q55

At 37°C , the osmotic pressure of blood is 8.21 atm. The amount of glucose that should be used per litre for an
intravenous injection so that it becomes isotonic with blood is (mark answer to nearest integer in grams)

Answer: 58

Solution:
π 8.21
C = =
RT 0.0821×310

W 8.21
=
180×1 0.0821×310

www.mathongo.com
AYJR 2024 (January) – Evening Shift Are You JEE Ready (AYJR)

Questions with Answer Keys and Solutions MathonGo


8.21 × 180
W = = 58.06 g
0.0821 × 310

Q56

How many of these elements have more first ionization energy then boron
Li, Be, C, N, O, F, Ne .

Answer: 6

Solution:
Here the order of first ionization energy is as follows
Ne > F > N > O > C > Be > B > Li

Hence six elements have more first ionization energy then boron.

Q57

The standard enthalpy of formation of NH3 is - 46.0 kJ/mol. If bond enthalpy of H2 is - 436 kJ/mol and that of N2 is -
712 kJ/mol , the average bond enthalpy of N - H bond in NH3 is : (mark answer to nearest integer in KJ/mol)

Answer: 352

Solution:
For The Reaction
1 3 ∘ −1
N2 (g)+ H2 (g)→ NH3 (g); ΔH = −46 KJ mol
f
2 2


ΔH = −46 = ∑ B.E − ∑ B.E
f

Reactants Product
∑ B.E = ∑ B.E + 46

Product Reactants
1 3
= (712)+ (436)+46
2 2

= 356 + 654 + 46 = 1056


Averge B.E for N-H Bond = 1056

3
= 352 KJ mol
−1

Q58

How many of the following statements is/are correct?


(A) The order of splitting energy is PtCl 2−

4
> PdCl
2−

4
> NiCl
2−

4
(consider only magnitude)
(B)[Ni(CO)4 ] is diamagnatic whereas [Ni(H 2
O)6 ]
2+
is paramagnetic.
(C)[Ni(CN)4 ]
2−
→ dsp
2
hybridized and paramagnetic.
(D) The magnetic moment of K 3 [Fe(CN)6 ] is √3 B.M

www.mathongo.com
AYJR 2024 (January) – Evening Shift Are You JEE Ready (AYJR)

Questions with Answer Keys and Solutions MathonGo


Answer: 3

Solution:
(a) Splitting energy increases down the group of transition metals complexes with same ligands.
(b) [Ni(CO) 4
]→ sp
3

2+ 2
3
[Ni(H O) ] → sp d
2 6

(c) [Ni(CN) 4]
2−
→ dsp
2
and diamagnetic
(d) for K 3 [Fe(CN)
6
]; M = √3

Q59

The degree of dissociation of I molecule at 1000 2



C and under 1.0 atmospheric pressure is 40% by volume. If the
dissociation is reduced to 20% at the same temperature, then if the total equilibrium pressure on the gas is 4.57 × 10 x

atm. Find x?

Answer: 0

Solution:

2
4α P
KP =
2
1 − α

α = 0.4, p = 1 atm

4 × 0.16 4 × 0.04
= Pnew
0.84 0.96

Pnew = 4.57 atm

Q60

Number of electrons having l + m = 0 in Mn(z = 25) is (l represent angular quantum number and m represent
magnetic quantum number)

Answer: 13

Solution:
2 2 6 2 6 2 5
Mn = 1s 2s 2p 3s 3p 4s 3d

Total 13 e −

www.mathongo.com
AYJR 2024 (January) – Evening Shift Are You JEE Ready (AYJR)

Questions with Answer Keys and Solutions MathonGo


Q61

If the mean deviation about the median of the numbers a, 2a, … . , 50a is 50, then |a| equals

(1) 4 ✅
(2) 5

(3) 2

(4) 3

Answer: (1)

Solution:

Median is the mean of 25 and 26 observation.


th th

25a+26a
M = = 25. 5a
2

Deviation is ∑(x i − M )=|25 .5|a|−|a||+|25 .5|a|−|2a||+. .. +|25 .5|a|−50|a||

∑(xi − M )= 2|a|(0 .5 +1 .5 +. .. + 24 .5)

∑ | xi −M |

Mean Deviation = N

1
⇒ 50 = {2|a|×(0 ⋅ 5 + 1 ⋅ 5+. . . +24 ⋅ 5)}
50

25
⇒ 2500 = 2|a|⋅ ⋅ 25
2

∴ |a|= 4

Q62

Number of points where the function f (x) = max(| tan x|, cos |x|) is non differentiable in the interval (−π, π) is

(1) 4 ✅
(2) 6

(3) 3

(4) 2

Answer: (1)

www.mathongo.com
AYJR 2024 (January) – Evening Shift Are You JEE Ready (AYJR)

Questions with Answer Keys and Solutions MathonGo


Solution:
The function is not differentiable and continuous at two points between x = −π/2&x = π/2 also function is not
continuous at x = π

2
and x = − π

hence at four points function is not differentiable

Q63

The number of ways of arranging the letters of the word NALGONDA, such that the letters of the word GOD occur in
that order (G before O and O before D), is

(1) 1250

(2) 1440

(3) 1560

(4) 1680 ✅
Answer: (4)

Solution:
8
The letters G, O, D can be arranged in ( ) = 56 ways.
3

The letters AANNL can be arranged in ways


5!
= 30
2!2!

∴ The desired number is 56 × 30 = 1680.

Q64

→ → → →
→ → → → → →
Let a , b, c be three non-zero vectors satisfying a = b × c + 2b where | b | = | c | = 2 and | a | ≤ 4. The sum

→ →
of possible value(s) of |2 a + b + c | is:

www.mathongo.com
AYJR 2024 (January) – Evening Shift Are You JEE Ready (AYJR)

Questions with Answer Keys and Solutions MathonGo


(1) 8

(2) 12

(3) 20 ✅
(4) 32

Answer: (3)

Solution:
→ →
→ →
a = b × c + 2b

Taking dot product with b
→ → → →
→ 2 → 2
a ⋅ b = 2| b | ⇒ | a || b | cos θ = 2| b |

4 → ∘
⇒ cos θ = ⇒ | a | = 4 ⇒ θ = 0

| a |



⇒ a = 2b
→ → →
→ → →
Now, b × c = 0 ⇒ b = c or b = − c

∴ Required sum = 12 + 8 = 20

Q65

Let f : [0, 2] → R be a function which is continuous on [0, 2] and is differentiable on (0, 2) with f (0) = 1. Let
2

for x ∈ [0, 2]. If F for all x ∈ (0, 2), then F (2) equals
x
′ ′
F (x) = ∫ f (√t)dt (x) = f (x)
0

(1) e 2
− 1

(2) e 4
− 1 ✅
(3) e − 1

(4) e 4

Answer: (2)

Solution:

www.mathongo.com
AYJR 2024 (January) – Evening Shift Are You JEE Ready (AYJR)

Questions with Answer Keys and Solutions MathonGo


2
x

F (x) = ∫ f (√t)dt for x ∈ [0, 2]


0


⇒ F (x) = f (x)2x
′ ′
Now F (x) = f (x)∀x ∈ (0, 2)


⇒ f (x) ⋅ 2x = f (x)


f (x)
⇒ = 2x
f (x)

2
⇒ In f (x) = x + c
2 2
x ∗c x ε
⇒ f (x) = e = e ⋅ e

As f (0) = 1 ⇒ 1 = e ε

2
x
∴ f (x) = e
2 2

So F (x) = ∫
x x x
e dx = e − 1
0

4
∴ F (2) = e − 1

Q66

4 2
x 2 4 x
∫ e (1 + x + 2x ) d (e ) = f (x) + C

where C is constant of integration and f (0) = 0. Then the value of f (1) + f ′


(0) is equal to

(1) e

(2) e 2

(3) 0

(4) e 3

Answer: (2)

Solution:

4 2
x 2 4 x
= ∫ e (1 + x + 2x ) e 2xdx

4 2
x +x 3 5
= 2∫ e (x + x + 2x ) dx

4 2 4 2
x +x 2 x +x 3
= 2∫ e xdx + ∫ x ⋅ e × (2x + 4x ) dx
4 2 4 2 4 2
x +x 2 x +x x +x
= 2∫ e xdx + x e − ∫ 2xe dx

4 2
2 x +x
= x e + c

Here
4 2
2 x +x
f (x) = x e

′ 2
f (1) + f (0) = e

www.mathongo.com
AYJR 2024 (January) – Evening Shift Are You JEE Ready (AYJR)

Questions with Answer Keys and Solutions MathonGo


Q67

Let PM be the perpendicular from the point P(1, 2, 3) to x − y plane. If OP makes an angle θ with the positive
direction of z-axis and OM makes an angle ϕ with the positive direction of x-axis, where O is the origin and θ and ϕ
are acute angles, then select incorrect option.

√5
(1) tan θ = 3

(2) sin θ sin ϕ = 2

√14

(3) tan ϕ = 2

(4) cos θ cos ϕ = 1

√14

Answer: (4)

Solution:
If P be (x, y, z), then from the figure,
x = r sin θ cos ϕ, y = r sin θ sin ϕ and z = r cos θ

1 = r sin θ cos ϕ, 2 = r sin θ sin ϕ and 3 = r cos θ

2 2 2 2
⇒ 1 + 2 + 3 = r or r = ±√14

1
∴ sin θ cos ϕ = , sin θ sin ϕ
√14

2 3
= and cos θ =
√14 √14

(neglecting negative sign as θ and ϕ are acute)


sin θ sin ϕ 2
=
sin θ cos ϕ 1

√5
Also, tan θ = 3

Q68

2 2
y
Consider the ellipse where f (x) is a positive decreasing function. The number of integral
x
+ = 1
2 f (k+12)
f (k − 4k + 6)

non-negative values of k for which major axis lies on the line y = 0 is

(1) 2

www.mathongo.com
AYJR 2024 (January) – Evening Shift Are You JEE Ready (AYJR)

Questions with Answer Keys and Solutions MathonGo


(2) 3

(3) 6 ✅
(4) 4

Answer: (3)

Solution:
2
f (k – 4k + 6) > f (k + 12)

2
⇒ k – 4k + 6 < k + 12

2
⇒ k – 5k– 6 < 0

⇒ (k– 6) (k + 1) < 0

⇒ k ∈ (– 1, 6)

Q69

If sin −1
x + sin
−1
y + sin
−1
z =

2
, then (x + y + z) 2

(1) 0

(2) 1

(3) 4

(4) 9 ✅
Answer: (4)

Solution:
The equation is true only if x = y = z = 1
2 2
(x + y + z) = 3 = 9.

Q70

The number of irrational roots of x 4


− 6x
3
+ 10x
2
− 6x + 1 = 0 is

(1) 0

(2) 1

(3) 2 ✅
(4) 4

www.mathongo.com
AYJR 2024 (January) – Evening Shift Are You JEE Ready (AYJR)

Questions with Answer Keys and Solutions MathonGo


Answer: (3)

Solution:
x
4
− 6x
3
+ 10x
2
− 6x + 1 = 0 . Dividing by x 2
,x
2
− 6x + 10 −
6

x
+
1

2
= 0
x

2 1
t − 2 − 6t + 10 = 0, t = x +
x

2 1
t − 6t + 8 = 0 ⇒ t = 2, 4 ⇒ x + = 2 ⇒ x = 1, 1
x

x +
1

x
= 4 ⇒ x
2
− 4x + 1 = 0 ⇒ x = 2 ± √3 .

Q71

If f (x) is the least degree polynomial such that f (n) = 1


, n = 1, 2, 3, 4, 5 , then f (0) =


n

(1) 137

60

(2)
97

60

(3) 119

60

(4) 5

Answer: (1)

Solution:
xf (x) − 1 = 0 has roots 1, 2, 3, 4, 5
∴ xf (x) − 1 = A(x − 1)(x − 2)(x − 3)(x − 4)(x − 5)

1
x = 0 ⇒ A =
120

1
∴ xf (x) = 1 + (x − 1)(x − 2)(x − 3)(x − 4)(x − 5)
120
5 4
x − 15x + … + kx 1
4 3
= or f (x) = (x − 15x + … + k)
120 120

where k = 120 (−1 + 1

2
+
1

3
+
1

4
+
1

5
)

k 1 1 1 1 137
∴ f (0) = = 1 + + + + =
120 2 3 4 5 60

Q72

A fair coin is tossed until one of the two sides occurs twice in a row. Then the Probability that number of tosses
required is even is

(1) 1

(2) 1


2

(3) 2

www.mathongo.com
AYJR 2024 (January) – Evening Shift Are You JEE Ready (AYJR)

Questions with Answer Keys and Solutions MathonGo


(4) 1

Answer: (3)

Solution:

P (E) = P {H H + H T H H + H T H T H H + … …}

+ P {T T + T H T T + T H T H T T + … … . . }

1 1 1
= 2{ + + + … … . . . ∞} = 2/3
2 3
4 4 4

Q73

Let a be a complex number such that |a| = 1. If the equation az 2


+ z + 1 = 0 has a pure imaginary root, then
tan(arg a) =

√5−1
(1) 2

√5+1
(2) 2

√5−1
(3) √ 2

(4) √
√5+1

2

Answer: (4)

Solution:
|a| = 1 ⇒ a = cos α + i sin α

The equation is (cos α + i sin α)z 2


+ z + 1 = 0 ⇒ z = iy is a root, y ∈ R
2
−(cos α + i sin α)y + iy + 1 = 0

Real parts ⇒ y 2
cos α = 1 , imaginary parts ⇒ y 2
sin α = y, y ≠ 0 ⇒ y sin α = 1

√5−1
Solving, cos α = 2
2
√5+1 √5+1 2√5+2 √5+1
2 2 2
⇒ sec α = = , tan α = sec α − 1 = ( ) − 1 = =
√5−1 2 2 4 2

Q74

Let I be the set of positive integers. R is a relation on the set I given by R = {(a, b) ∈ I × I ∣ log (
2
a
) is a non-
b

negative integer}, then R is

(1) neither symmetric nor transitive but reflexive.

(2) reflexive, transitive but not symmetric ✅


www.mathongo.com
AYJR 2024 (January) – Evening Shift Are You JEE Ready (AYJR)

Questions with Answer Keys and Solutions MathonGo


(3) neither reflexive nor transitive but symmetric

(4) equivalence relation.

Answer: (2)

Solution:
aRb ⇔ a = 2
k
⋅ a it is true for k = 0
∴ reflexive (2, 1) ∈ R but (1, 2) ∉ R ⇒ it is not symmetric
if a = 2 k1
b and b = 2 k2
c , then a = 2 k1 +k2
c

⇒ it is transistive.

Q75

Let a, b, c, d be four positive integers in arithemetic progression such that a < b < c < d and ab = c + d − 1. The sum
of all possible value(s) of ' a ' is

(1) 6.5

(2) 7 ✅
(3) 13

(4) 6

Answer: (2)

Solution:
Let common difference be λ.
⇒ a(a + λ) = 2a + 5λ − 1

16
⇒ λ = − a − 3
5 − a

possible values of 'a' are 3 and 4.

Q76

A line cuts the x-axis at A(7, 0) and the y-axis at B(0, −5), a variable line P Q is drawn perpendicular to AB cutting
the x-axis at P and y-axis at Q. If AQ and BP intersect at R then locus of R is

(1) x 2
+ y
2
− 7x + 5y = 0 ✅
(2) x 2
+ y
2
+ 7x − 5y = 0

(3) x 2
− y
2
− 7x + 5y = 0

www.mathongo.com
AYJR 2024 (January) – Evening Shift Are You JEE Ready (AYJR)

Questions with Answer Keys and Solutions MathonGo


(4) x 2
− y
2
+ 7x − 5y = 0

Answer: (1)

Solution:
Note the AR will be perpendicular to BP. Let R be (h, k)

k + 5 k − 0
( )( ) = −1
h − 0 h − 7

2 2
⇒ k + 5k = −h + 7h

∴ Locus of R is x 2
+ y
2
− 7x + 5y = 0

Q77

The radius of the circle whose centre is (−8, 0) and which cuts the parabola y 2
= 8x at A & B such that the common
¯¯¯¯¯¯¯¯
chord AB subtends a right angle at the vertex of the parabola, is

(1) 20

(2) 20√5

(3) 8√5 ✅
(4) 40√2

Answer: (3)

Solution:
Clearly common chord AB is double ordinate of parabola
So, A = (2t 2
1
2
, 4t1 ) ⇒ B = (2t , −4t1 )
1

As △AOB = 90 , we have t ∘
2 = −
4

t1

−4 2
∴ −t = ⇒ t = 4 ⇒ t1 = ±2
t1 1

ie A = (8, 8), B = (8, −8)

If C = (−8, 0) , radius = CA = √256 + 64 = √320 = 8√5

Q78

www.mathongo.com
AYJR 2024 (January) – Evening Shift Are You JEE Ready (AYJR)

Questions with Answer Keys and Solutions MathonGo


Let M be a square matrix of order 3 such that MM T
= I and M 2
= I . Also M −1
+ adj(M) = 0 , if P is another matrix
such that P + 2M = 0 then value of det(PP T
P
−1
)

(1) 4

(2) 16

(3) 24

(4) 8 ✅
Answer: (4)

Solution:

T 2
MM = I and M = I
−1
M + adj(M) = 0

−1
M = − adj(M)

−1
∣M ∣ = | − adj(M)|
∣ ∣

3
= (−1) | adj M|

1 2
= −|M|
|M|

3
∴ |M| = −1 OR |M| = −1

NowP + 2M = 0 ∴ P = −2M

|P| = | − 2M|

3
= (−2) ⋅ |M|

= −8 × −1

|P| = 8

T −1
1
∣PP P ∣ = |P|P ∣ ⋅ = 8
∣ ∣
|P|

Q79

Area bounded between the curves y = √4 − x and y 2 2


= 3|x| is :

(1)
π−1

√3

(2)
2π−1

3√3

(3)
2π−√3

3

(4) none of these

Answer: (3)

www.mathongo.com
AYJR 2024 (January) – Evening Shift Are You JEE Ready (AYJR)

Questions with Answer Keys and Solutions MathonGo


Solution:

1
1/2
x 4 x √3 ⋅ 2x
2 −1
= 2( √4 − x + sin ( ) − )
2 2 2 3
0

2π − √3
=
3

Q80

2 [x]

−1
[
2
] dx , (where [.] denotes the greatest integer function) is equal to
1+x

(1) -2

(2) -1 ✅
(3) zero

(4) 1

Answer: (2)

Solution:
[x] = 0, ∀x ∈ [0, 1)

For x ∈ [1, 2), [x] = 1


[x] [x]
1
⇒ = < 1, ∀x ∈ [1, 2) ⇒ [ ] = 0
2 2 2
1+x 1+x 1+x

[x]
For x ∈ [−1, 0), [x] = −1 ⇒ 2
= −
1

2
1+x 1+x

Clearly, 2 ≥ 1 + x 2
> 1, ∀x ∈ [−1, 0)

1 1 1 1
≤ < 1 ⇒ − ≥ − > −1
2 2
2 1 + x 2 1 + x

[x]
⇒ [ ] = −1∀x ∈ [−1, 0)
2
1 + x
0
Thus, the given integral = − ∫ −1
dx = −1

www.mathongo.com
AYJR 2024 (January) – Evening Shift Are You JEE Ready (AYJR)

Questions with Answer Keys and Solutions MathonGo


Q81

Let a, b are two integers such that 0 < a < b < 10 and arithmetic mean of a and b is exactly 8 more than its
6

geometric mean. If number of such ordered pairs is N, then N is equal to

Answer: 995

Solution:

a + b
= √ab + 8
2

⇒ a + b − 2√ab = 16

3
⇒ √b − √a = 4, 0 < √a, √b < 10

so if
√b = 999 ⇒ √a = 995

√b = 998 ⇒ √a = 994

√b = 5 ⇒ √a = 1

Therefore 995 such pairs are there

Q82

Let f (x) = 2x 3
− 3(2 + p)x
2
+ 12px + ln(16 − p )
2
. If f (x) has exactly one local maxima and one local minima, then
the number of integral values of p is .....

Answer: 6

Solution:
We have, f (x) = 2x 3
− 3(2 + p)x
2 2
+ 12px + ln(16 − p )

Now, f ′
(x) = 6 (x
2
− (2 + p)x + 2p)


∴ f (x) = 0 ⇒ x = 2, p

As, f (x) has exactly one local maxima and one local minima
So, p ≠ 2
Also, 16 − p 2
> 0

\(\begin{aligned}
& \Rightarrow p^2-16<0 \\
& \Rightarrow(p+4)(p-4)<0 \\
& \Rightarrow-4\end{aligned}\)
∴ The possible integral values of p are −3, −2, −1, 0, 1, 3

Q83

www.mathongo.com
AYJR 2024 (January) – Evening Shift Are You JEE Ready (AYJR)

Questions with Answer Keys and Solutions MathonGo


Number of integral points (points whose abscissa & ordinate both are integer) in the common region bounded by
and Re( , is
z+1 1 1
∣ ∣
⩾ 1 ) ⩾
∣ z−1 ∣ z 2

Answer: 3

Solution:

1 1
Re( ) ⩾
x + iy 2

x − iy 1
Re( ) ⩾
x
2
+ y
2 2

2 2
⇒ 2x ⩾ x + y

∵ z ≠ 0, 1

∴ 3 integral points.

Q84

dy y y
The solution of x 2

dx
− xy = 1 + cos
x
is tan( px
) = c −
1

2
where c is constant then, find the value of p + q ?
qx

Answer: 4

Solution:

www.mathongo.com
AYJR 2024 (January) – Evening Shift Are You JEE Ready (AYJR)

Questions with Answer Keys and Solutions MathonGo


dy y
2
x − xy = 1 + cos( )
dx x

x(xdy − ydx) y
or = 1 + cos( )
dx x
(xdy−ydx)

x
2 dx
⇒ =
y 3
x
1 + cos( )
x

y
d( )
x
dx
⇒ ∫ = ∫
y 3
x
1 + cos( )
x

y
d( )
1 x dx
⇒ ∫ = ∫
2 2
y
x
3
cos ( )
2x

y
tan( ) −2
1 2x x
⇒ = + c
2 1 −2
2

y 1
⇒ tan( ) + = c
2
2x 2x

p = q = 2

Q85

If a line passing through (2, 1, 4) cuts off an intercept of minimum length between two non coplanar lines
y y
x − 6 =
α
= −z and x = 0
= z , then α is equal to

Answer: 2

Solution:
Line passes through P(2, 1, 4) lie along shortest distance between two non-coplanar lines
x − 6 y z
L1 = = = = k1 ⇒ A (6 + k1 , αk1 , −k1 )
1 x −1
x y z
L2 = = = = k2 ⇒ B (k2 , 0, k2 )
1 0 1

PB perpendicular to L 2 ⇒ k2 = 3

PA perpendicular to L 1

2
⇒ (6 + k1 − 2) 1 + (αk1 − 1) ⋅ α + (−k1 − 4) (−1) = 0 α k1 − α + 2k1 + 8 = 0 …

AB perpendicular to L 1

www.mathongo.com
AYJR 2024 (January) – Evening Shift Are You JEE Ready (AYJR)

Questions with Answer Keys and Solutions MathonGo


⇒ (6 + k1 − 3) + α (αk1 ) + (−1)(−k, −3) = 0

2
α k1 + 2k1 + 6 = 0 … (2)

(1) − (2) ⇒ α = 2

Q86

In how many ways Ram can distribute 40 apples in his six children named A, B, C, D, E and F such that A gets two
more than B, C gets 3 more than F and D gets five less than E and every one must have atleast one fruit

Answer: 91

Solution:
Let n(B) = x n(A) = x + 2

n(F ) = y n(C) = y + 3

n(D) = z n(E) = z + 5

x + y + z + x + 2 + y + 3 + z + 5 = 40

x + y + z = 15 but x ≥ 1. y ≥ 1 and z ≥ 1
So total ways of distribution is 14
C2 = 91

Q87

1 0 0
⎡ ⎤

Let P = ⎢4 1 0⎥ and I be the identity matrix of order 3. If Q = [q ij


] is a matrix such that P 500
−Q = I , then
⎣ ⎦
8 4 1
q +q
31

q
32
equals
21

Answer: 1001

Solution:

www.mathongo.com
AYJR 2024 (January) – Evening Shift Are You JEE Ready (AYJR)

Questions with Answer Keys and Solutions MathonGo


0 0 0
⎡ ⎤

P = I + B, B = ⎢ 4 0 0⎥
⎣ ⎦
8 4 0

4 0 0 0 0 0 0 0 0
⎡ ⎤⎡ ⎤ ⎡ ⎤
2
B = ⎢4 0 0⎥⎢4 0 0⎥ = ⎢0 0 0⎥
⎣ ⎦⎣ ⎦ ⎣ ⎦
8 4 0 8 4 0 16 0 0

3
B = 0
500 500 2
P = (I + B) = I + 500B + 250 × 499B

500 2
⇒ P − I = 500 B + 250 × 499B

q31 + q32

q21

500 × 8 + 250 × 499 × 16 + 500 × 4


=
500 × 4 + 0

= 1001

Q88

128

The remainder when 16 32


is divided by 7 is

Answer: 2

Solution:

128
128 5 640
Let N = 32 = (2 ) = 2

320
= 4 = 3 m + 1

3 m+1 12 m+4 12 m+3 4 m+1


(16) = 2 = 2.2 = 2 (8 ) = 2(7p + 1)

Remainder is 2

Q89

Let A = {1, 2, 3, 4} and B = {0, 1, 2, 3, 4, 5}. The number of one-one functions from A to B which are not increasing
is

Answer: 345

Solution:
|A| = 4, |B| = 6 , the number of one-one functions is 6
P4 = 360 .
6
The number of increasing functions is ( ) = 15 .
4

The desired number is 360 − 15 = 345

Q90

www.mathongo.com
AYJR 2024 (January) – Evening Shift Are You JEE Ready (AYJR)

Questions with Answer Keys and Solutions MathonGo


If P 1, P2 , P3 are the points on ellipse 3x 2
+ y
2
− 12 = 0 and P 1
′ ′
P2 P3 ' are their corresponding points on the auxillary
circle, then the area of triangle P 1
′ ′
P2 P3 ' is λ times the area of triangle P 1 P2 P3 , then λ is
2

Answer: 3

Solution:
Let P 1
(2 cos θ1 2√3 sin θ1 )

1 ′ ′
P (2√3 cos θ , 2√3 sin θ1 )
1 1

∣ 2√3 cos θ 2√3 sin θ1 1∣


1
∣ ∣
Area P 1
1
P
1
2
P
1
3
=
1

2
∣ 2√3 cos θ
2 2√3 sin θ2 1∣
∣ ∣
∣ 2√3 cos θ3 2√3 sin θ3 1∣

∣ 2 cos θ 2√3 sin θ1 1∣


1
1 ∣ ∣
P1 P2 P3 = ∣ 2 cos θ2 2√3 sin θ2 1∣
2 ∣ ∣
∣ 2 cos θ3 2√3 sin θ3 1∣

2√ 3 ⋅ 2√ 3
2
λ = = √3 ⇒ λ = 3
2.2√3

www.mathongo.com

You might also like